Open questions about posets












28












$begingroup$


Partially ordered sets (posets) are important objects in combinatorics (with basic connections to extremal combinatorics and to algebraic combinatorics) and also in other areas of mathematics. They are also related to sorting and to other questions in the theory of computing. I am asking for a list of open questions and conjectures about posets.










share|cite|improve this question











$endgroup$








  • 3




    $begingroup$
    Union closed sets conjecture of Frankl. Gerhard "Master Of The Obvious, Apparently" Paseman, 2019.02.06.
    $endgroup$
    – Gerhard Paseman
    Feb 6 at 17:56






  • 3




    $begingroup$
    I have voted to close on the grounds the question is too broad. It seems any open problem related to sorting, searching, chains, antichains, cardinals, ordinals, surreal numbers, least fixed points, ... might be relevant.
    $endgroup$
    – Mark Wildon
    Feb 6 at 19:16








  • 5




    $begingroup$
    To be clear: The questions need to be questions about posets. For example, Frankl's union closed set conjecture is a great problem but it is outside the scope of the question as I see it. Gil, "if in doubt, make it a comment rather than an answer" Kalai.
    $endgroup$
    – Gil Kalai
    Feb 6 at 19:37








  • 5




    $begingroup$
    Here is the list from Ivan Rival that Gil Kalai mentioned: link.springer.com/content/pdf/10.1007%2FBF00334860.pdf. The last problem is interesting: I think it is saying "determine the Dedekind numbers (en.wikipedia.org/wiki/Dedekind_number)"; but it's too much to hope for an exact formula, right?
    $endgroup$
    – Sam Hopkins
    Feb 6 at 19:48






  • 7




    $begingroup$
    Frankl's conjecture can be reformulated in terms of posets. See Enumerative Combinatorics, vol. 1, 2nd ed., Exercise 3.102. Namely, let $L$ be a finite $n$-element lattice. Does there exist a join-irreducible $t$ of $L$ such that the principal dual order ideal $V_t:={ sin Lcolon sgeq t}$ has at most $n/2$ elements?
    $endgroup$
    – Richard Stanley
    Feb 7 at 14:28
















28












$begingroup$


Partially ordered sets (posets) are important objects in combinatorics (with basic connections to extremal combinatorics and to algebraic combinatorics) and also in other areas of mathematics. They are also related to sorting and to other questions in the theory of computing. I am asking for a list of open questions and conjectures about posets.










share|cite|improve this question











$endgroup$








  • 3




    $begingroup$
    Union closed sets conjecture of Frankl. Gerhard "Master Of The Obvious, Apparently" Paseman, 2019.02.06.
    $endgroup$
    – Gerhard Paseman
    Feb 6 at 17:56






  • 3




    $begingroup$
    I have voted to close on the grounds the question is too broad. It seems any open problem related to sorting, searching, chains, antichains, cardinals, ordinals, surreal numbers, least fixed points, ... might be relevant.
    $endgroup$
    – Mark Wildon
    Feb 6 at 19:16








  • 5




    $begingroup$
    To be clear: The questions need to be questions about posets. For example, Frankl's union closed set conjecture is a great problem but it is outside the scope of the question as I see it. Gil, "if in doubt, make it a comment rather than an answer" Kalai.
    $endgroup$
    – Gil Kalai
    Feb 6 at 19:37








  • 5




    $begingroup$
    Here is the list from Ivan Rival that Gil Kalai mentioned: link.springer.com/content/pdf/10.1007%2FBF00334860.pdf. The last problem is interesting: I think it is saying "determine the Dedekind numbers (en.wikipedia.org/wiki/Dedekind_number)"; but it's too much to hope for an exact formula, right?
    $endgroup$
    – Sam Hopkins
    Feb 6 at 19:48






  • 7




    $begingroup$
    Frankl's conjecture can be reformulated in terms of posets. See Enumerative Combinatorics, vol. 1, 2nd ed., Exercise 3.102. Namely, let $L$ be a finite $n$-element lattice. Does there exist a join-irreducible $t$ of $L$ such that the principal dual order ideal $V_t:={ sin Lcolon sgeq t}$ has at most $n/2$ elements?
    $endgroup$
    – Richard Stanley
    Feb 7 at 14:28














28












28








28


13



$begingroup$


Partially ordered sets (posets) are important objects in combinatorics (with basic connections to extremal combinatorics and to algebraic combinatorics) and also in other areas of mathematics. They are also related to sorting and to other questions in the theory of computing. I am asking for a list of open questions and conjectures about posets.










share|cite|improve this question











$endgroup$




Partially ordered sets (posets) are important objects in combinatorics (with basic connections to extremal combinatorics and to algebraic combinatorics) and also in other areas of mathematics. They are also related to sorting and to other questions in the theory of computing. I am asking for a list of open questions and conjectures about posets.







co.combinatorics computer-science open-problems order-theory posets






share|cite|improve this question















share|cite|improve this question













share|cite|improve this question




share|cite|improve this question








edited Feb 8 at 9:04


























community wiki





6 revs, 3 users 92%
Gil Kalai









  • 3




    $begingroup$
    Union closed sets conjecture of Frankl. Gerhard "Master Of The Obvious, Apparently" Paseman, 2019.02.06.
    $endgroup$
    – Gerhard Paseman
    Feb 6 at 17:56






  • 3




    $begingroup$
    I have voted to close on the grounds the question is too broad. It seems any open problem related to sorting, searching, chains, antichains, cardinals, ordinals, surreal numbers, least fixed points, ... might be relevant.
    $endgroup$
    – Mark Wildon
    Feb 6 at 19:16








  • 5




    $begingroup$
    To be clear: The questions need to be questions about posets. For example, Frankl's union closed set conjecture is a great problem but it is outside the scope of the question as I see it. Gil, "if in doubt, make it a comment rather than an answer" Kalai.
    $endgroup$
    – Gil Kalai
    Feb 6 at 19:37








  • 5




    $begingroup$
    Here is the list from Ivan Rival that Gil Kalai mentioned: link.springer.com/content/pdf/10.1007%2FBF00334860.pdf. The last problem is interesting: I think it is saying "determine the Dedekind numbers (en.wikipedia.org/wiki/Dedekind_number)"; but it's too much to hope for an exact formula, right?
    $endgroup$
    – Sam Hopkins
    Feb 6 at 19:48






  • 7




    $begingroup$
    Frankl's conjecture can be reformulated in terms of posets. See Enumerative Combinatorics, vol. 1, 2nd ed., Exercise 3.102. Namely, let $L$ be a finite $n$-element lattice. Does there exist a join-irreducible $t$ of $L$ such that the principal dual order ideal $V_t:={ sin Lcolon sgeq t}$ has at most $n/2$ elements?
    $endgroup$
    – Richard Stanley
    Feb 7 at 14:28














  • 3




    $begingroup$
    Union closed sets conjecture of Frankl. Gerhard "Master Of The Obvious, Apparently" Paseman, 2019.02.06.
    $endgroup$
    – Gerhard Paseman
    Feb 6 at 17:56






  • 3




    $begingroup$
    I have voted to close on the grounds the question is too broad. It seems any open problem related to sorting, searching, chains, antichains, cardinals, ordinals, surreal numbers, least fixed points, ... might be relevant.
    $endgroup$
    – Mark Wildon
    Feb 6 at 19:16








  • 5




    $begingroup$
    To be clear: The questions need to be questions about posets. For example, Frankl's union closed set conjecture is a great problem but it is outside the scope of the question as I see it. Gil, "if in doubt, make it a comment rather than an answer" Kalai.
    $endgroup$
    – Gil Kalai
    Feb 6 at 19:37








  • 5




    $begingroup$
    Here is the list from Ivan Rival that Gil Kalai mentioned: link.springer.com/content/pdf/10.1007%2FBF00334860.pdf. The last problem is interesting: I think it is saying "determine the Dedekind numbers (en.wikipedia.org/wiki/Dedekind_number)"; but it's too much to hope for an exact formula, right?
    $endgroup$
    – Sam Hopkins
    Feb 6 at 19:48






  • 7




    $begingroup$
    Frankl's conjecture can be reformulated in terms of posets. See Enumerative Combinatorics, vol. 1, 2nd ed., Exercise 3.102. Namely, let $L$ be a finite $n$-element lattice. Does there exist a join-irreducible $t$ of $L$ such that the principal dual order ideal $V_t:={ sin Lcolon sgeq t}$ has at most $n/2$ elements?
    $endgroup$
    – Richard Stanley
    Feb 7 at 14:28








3




3




$begingroup$
Union closed sets conjecture of Frankl. Gerhard "Master Of The Obvious, Apparently" Paseman, 2019.02.06.
$endgroup$
– Gerhard Paseman
Feb 6 at 17:56




$begingroup$
Union closed sets conjecture of Frankl. Gerhard "Master Of The Obvious, Apparently" Paseman, 2019.02.06.
$endgroup$
– Gerhard Paseman
Feb 6 at 17:56




3




3




$begingroup$
I have voted to close on the grounds the question is too broad. It seems any open problem related to sorting, searching, chains, antichains, cardinals, ordinals, surreal numbers, least fixed points, ... might be relevant.
$endgroup$
– Mark Wildon
Feb 6 at 19:16






$begingroup$
I have voted to close on the grounds the question is too broad. It seems any open problem related to sorting, searching, chains, antichains, cardinals, ordinals, surreal numbers, least fixed points, ... might be relevant.
$endgroup$
– Mark Wildon
Feb 6 at 19:16






5




5




$begingroup$
To be clear: The questions need to be questions about posets. For example, Frankl's union closed set conjecture is a great problem but it is outside the scope of the question as I see it. Gil, "if in doubt, make it a comment rather than an answer" Kalai.
$endgroup$
– Gil Kalai
Feb 6 at 19:37






$begingroup$
To be clear: The questions need to be questions about posets. For example, Frankl's union closed set conjecture is a great problem but it is outside the scope of the question as I see it. Gil, "if in doubt, make it a comment rather than an answer" Kalai.
$endgroup$
– Gil Kalai
Feb 6 at 19:37






5




5




$begingroup$
Here is the list from Ivan Rival that Gil Kalai mentioned: link.springer.com/content/pdf/10.1007%2FBF00334860.pdf. The last problem is interesting: I think it is saying "determine the Dedekind numbers (en.wikipedia.org/wiki/Dedekind_number)"; but it's too much to hope for an exact formula, right?
$endgroup$
– Sam Hopkins
Feb 6 at 19:48




$begingroup$
Here is the list from Ivan Rival that Gil Kalai mentioned: link.springer.com/content/pdf/10.1007%2FBF00334860.pdf. The last problem is interesting: I think it is saying "determine the Dedekind numbers (en.wikipedia.org/wiki/Dedekind_number)"; but it's too much to hope for an exact formula, right?
$endgroup$
– Sam Hopkins
Feb 6 at 19:48




7




7




$begingroup$
Frankl's conjecture can be reformulated in terms of posets. See Enumerative Combinatorics, vol. 1, 2nd ed., Exercise 3.102. Namely, let $L$ be a finite $n$-element lattice. Does there exist a join-irreducible $t$ of $L$ such that the principal dual order ideal $V_t:={ sin Lcolon sgeq t}$ has at most $n/2$ elements?
$endgroup$
– Richard Stanley
Feb 7 at 14:28




$begingroup$
Frankl's conjecture can be reformulated in terms of posets. See Enumerative Combinatorics, vol. 1, 2nd ed., Exercise 3.102. Namely, let $L$ be a finite $n$-element lattice. Does there exist a join-irreducible $t$ of $L$ such that the principal dual order ideal $V_t:={ sin Lcolon sgeq t}$ has at most $n/2$ elements?
$endgroup$
– Richard Stanley
Feb 7 at 14:28










10 Answers
10






active

oldest

votes


















21












$begingroup$

The 1/3-2/3 conjecture is probably considered one of the most significant open problems about finite posets; see the Wikipedia page: https://en.wikipedia.org/wiki/1/3%E2%80%932/3_conjecture.






share|cite|improve this answer











$endgroup$





















    16












    $begingroup$

    Here's a problem that I believe has little chance of being resolved, and it's also not so clear to me what the motivation behind the problem is, but it involves some very pretty algebraic combinatorics.



    Stanley (in his PhD thesis) defined a poset $P$ to be Gaussian if for every $mgeq 0$ we have
    $$ sum_{I in J(Ptimes [m])} q^{#I} = frac{prod_{i=1}^{r}(1-q^{h_i+m})}{prod_{i=1}^{r}(1-q^{h_i})}$$
    where $r$ and $h_1,ldots,h_r$ are constants independent of $m$. (Here $J(P times [m])$ is the distributive lattice of order ideals of the Cartesian product of $P$ and the chain poset $[m]={1,2,ldots,m}$.)



    Using results from Standard Monomial Theory, in "Bruhat Lattices, Plane Partition Generating Functions, and Minuscule Representations", Proctor proved that if $P$ is a minuscule poset, then $P$ is Gaussian. Here a minuscule poset is a certain finite poset coming from a minuscule representation of a semisimple Lie algebra; these have been classified- see Proctor's paper for the list. Furthermore, Proctor conjectured that the minuscule posets are the only Gaussian posets.



    I think a few things are known (due to some mixture of Proctor and Stanley) about this conjecture: that a Gaussian poset $P$ is a disjoint union of connected Gaussian posets; that a connected Gaussian poset has to be graded; that $r$ has to be the number of elements of $P$; that the $h_i$ have to be the ranks of the elements $pin P$; maybe some more things too. But like I said I think the full conjecture is pretty hopeless.






    share|cite|improve this answer











    $endgroup$









    • 10




      $begingroup$
      I don't think that the definition of a Gaussian poset is due to Proctor.
      $endgroup$
      – Richard Stanley
      Feb 6 at 18:59






    • 3




      $begingroup$
      @RichardStanley: apologies, Richard!
      $endgroup$
      – Sam Hopkins
      Feb 6 at 19:00










    • $begingroup$
      @SamHopkins : Reading between the lines, I wonder if you suspect that the conjecture may be false but the only counterexamples are "sporadic"?
      $endgroup$
      – Timothy Chow
      Feb 7 at 21:12










    • $begingroup$
      @TimothyChow: I think it could be false and in that case perhaps a large computer search could turn up a counterexample. If it’s true, it might just be true “by accident,” which would make it very hard to prove.
      $endgroup$
      – Sam Hopkins
      Feb 7 at 21:16



















    11












    $begingroup$

    Here's another classic from algebraic combinatorics.



    In his PhD thesis "Ordered Structures and Partitions", Stanley introduces the $(P,omega)$-partition generating function of a labeled poset $(P,omega)$. This is defined to be $K(P,omega) := sum_{sigma in A^r(P,omega)} x^{sigma}$, where $A^r(P,omega)$ is the set of all reverse $(P,omega)$-partitions (i.e., fillings of the poset $P$ with entries in $mathbb{N}$ obeying certain inequalities depending on $omega$ and the order structure of $P$), and where we use the notation $x^f := prod_{i geq 1} x_i^{#f^{-1}(i)}$. Stanley shows that $K(P,omega)$ is always a quasisymmetric function. When $P$ is the poset of a skew Young diagram and $omega$ is a compatible ``Schur labelling'' then $K_{(P,omega)}$ is in fact a symmetric function (namely, a skew Schur funciton). Stanley's conjecture is that $K_{(P,omega)}$ is a symmetric function if and only if $(P,omega)$ is isomorphic to $(P_{lambda/mu},w)$, where $lambda/mu$ is a skew-shape and $w$ is a Schur labelling of $P_{lambda/mu}$. There has been some progress on this conjecture due to Malvenuto; I don't know the exact status of what has been proved.






    share|cite|improve this answer











    $endgroup$









    • 1




      $begingroup$
      For Malvenuto's result see link.springer.com/article/10.1007/BF01195328.
      $endgroup$
      – Richard Stanley
      Feb 8 at 1:22



















    6












    $begingroup$

    What about the Stanley–Stembridge conjecture on (3+1)-free posets? Given a poset $P$ with vertex set $V = {1,2,ldots,n}$, define the symmetric function $X_P$ in countably many indeterminates $x_1, x_2, ldots$ by
    $$X_P := sum_{kappa:Vtomathbb{N}} x_{kappa(1)}x_{kappa(2)}cdots x_{kappa(n)}$$
    where the sum is over all maps $kappa:V tomathbb{N}$ such that the pre-image $kappa^{-1}(i)$ of every $iinmathbb{N}$ is a totally ordered subset of $P$. Then the conjecture is that if $P$ is (3+1)-free (i.e., that $P$ contains no induced subposet isomorphic to the disjoint union of a 3-element chain and a 1-element chain) then the expansion of $X_P$ in terms of elementary symmetric functions has nonnegative coefficients.



    This conjecture grew out of certain positivity conjectures about immanants.
    Guay-Paquet has reduced the conjecture to the case of unit interval orders (i.e., posets that are both (3+1)-free and (2+2)-free), and the unit interval order case has a graded generalization (due to Shareshian and Wachs), which has close connections with representation theory and Hessenberg varieties.






    share|cite|improve this answer











    $endgroup$





















      6












      $begingroup$

      Let $P$ be a (not necessarily finite) poset such that each element covers and is covered by a finite number of elements. Then we can define the operators $U,Dcolon mathbb{Q}Ptomathbb{Q}P$ on the vector space of formal linear combinations of elements of $P$ by $U(p) = sum_{plessdot q} q$ and $D(p) = sum_{qlessdot p}q$. Such a poset $P$ is called $r$-differential if it is locally finite, graded, with a unique minimal element, and these up and down operators satisfy $DU-UD=rI$ (where $I$ is the identity map). See https://en.wikipedia.org/wiki/Differential_poset.



      The two prominent examples of $1$-differential posets are Young's lattice and the Young-Fibonacci lattice. It is known that these are the only $1$-differential lattices, although this was only proved relatively recently by Byrnes (https://conservancy.umn.edu/handle/11299/142992). The open problem is: are all $r$-differential lattices products of Young's lattice and the Young-Fibonacci lattice?






      share|cite|improve this answer











      $endgroup$













      • $begingroup$
        Artistic rendering of a differential lattice by Mrs. Marlena Hewitt: facebook.com/ARTMARLENA/photos/a.320409601327165/…
        $endgroup$
        – Tri
        Feb 9 at 23:39





















      4












      $begingroup$

      Let $f : mathbb{F}_2^n rightarrow mathbb{F}_2$ be a boolean function. Ordering $mathbb{F}_2$ so that $0<1$, we say that $f$ is monotone if $f(x_1,ldots,x_n) le f(y_1, ldots, y_n)$ whenever $x_1 le y_1, ldots, x_n le y_n$. The Dedekind number $M(n)$ is the number of monotone Boolean functions of $n$ variables, up to logical equivalence.



      Equivalently, $M(n)$ is the number of Boolean functions (up to logical equivalence) that can be expressed without negation, or the number of access structures on $n$ people. For example,



      $$(x_1 wedge x_2) vee (x_1 wedge x_3) vee (x_2 wedge x_3)$$



      is counted by $M(3)$, and corresponds to the access structure where any pair of people, or all three people, form an authorized set. In general the minimal authorized sets form an antichain in the poset of subsets of ${1,ldots, n}$, so $M(n)$ is the number the number of antichains in $mathcal{P}({1,ldots,n})$. Since the maximal subsets in an abstract simplicial complex form an antichain, $M(n)$ is also the number of abstract simplicial complexes on ${1,ldots, n}$.



      The antichain interpretation makes it plausible that



      $$log M(n) sim binom{n}{frac{n}{2}} sim sqrt{frac{2}{pi}} frac{2^n}{sqrt{n}} $$



      and this was proved by Kleitman.



      An important open problem (mentioned in the comments above) is to give a more precise asymptotic estimate for $M(n)$, and to determine the value of $M(n)$ for small $n$. At the moment, $M(n)$ is known precisely only for $n le 7$. The linked paper has the best known asymptotic formula, due to Korshunov.






      share|cite|improve this answer











      $endgroup$









      • 2




        $begingroup$
        Small correction: $M(8)$ is known, see OEIS sequence A000372 and Wiedemann, D.: A computation of the eighth Dedekind number (Order, March 1991, Vol. 8, Issue 1, pp 5-6).
        $endgroup$
        – nomadictype
        Feb 8 at 3:37










      • $begingroup$
        I have an English translation of Dr. Korshunov's hundred-page paper. (Even he does not have an English translation.)
        $endgroup$
        – Tri
        Feb 8 at 20:49










      • $begingroup$
        I believe it was Yamamoto proved that if n is even, then M(n) is even. What can we say about n odd? Using an idea I have long forgotten, I think Ciucu and Stembridge got some data for n up to 19, with one exception (ether n=11 or n=13).
        $endgroup$
        – Tri
        Feb 9 at 23:59



















      4












      $begingroup$

      The fish-scale conjecture: In every poset not containing an infinite antichain there exist a chain $C$ and a decomposition of the vertex set into antichains $A_i$, such that $Ccap A_i neq emptyset$ for all $iin I$. See Conjecture 10.1 in this article by Ron Aharoni and Eli Berger.



      Interestingly, this conjecture is implied by the following covering problem in hypergraphs: Let $H=(V,E)$ be a hypergraph with the following properties:





      1. $bigcup E = V$,

      2. Any subset of $ein E$ is a member of $E$, and

      3. whenever $Ssubseteq V$ and every $2$-element subset of $S$ belongs to $E$, then $Sin E$.


      Then there is $Jsubseteq E$ such that whenever $Ksubseteq E$ has the property that $bigcup K = V$ then $|Jsetminus K| leq |Ksetminus J|$. (In that case $J$ is said to be a "strongly minimal cover" because it covers $V$ "more efficiently" in some sense, than any other cover.)



      This covering problem is mentioned in Conjecture 10.3.(2) of the same paper.






      share|cite|improve this answer











      $endgroup$













      • $begingroup$
        I think the linked article might be wrong.
        $endgroup$
        – Sam Hopkins
        Feb 8 at 15:50










      • $begingroup$
        Why do you think that?
        $endgroup$
        – Dominic van der Zypen
        Feb 8 at 19:25










      • $begingroup$
        there doesn't seem to be a Conjecture 10.1 and there are 3 authors, not 2.
        $endgroup$
        – Sam Hopkins
        Feb 8 at 19:30






      • 1




        $begingroup$
        It looks like the correct paper is this one: math.haifa.ac.il/berger/menger_2nd_submition.pdf
        $endgroup$
        – Sam Hopkins
        Feb 8 at 20:22










      • $begingroup$
        Thanks for providing this link!
        $endgroup$
        – Dominic van der Zypen
        Feb 8 at 20:56



















      3












      $begingroup$

      Find a nice expression for the number of down-sets (order ideals) of a product of four finite chains (totally ordered sets), $|bf2^{(bf ktimes bf mtimes bf ntimes bf r)}|$, where $k,m,n,r$ are positive integers and "$bf k$" denotes the $k$-element chain, ${0,1,dots,k-1}$.



      Richard P. Stanley writes (on page 83 of Ordered Structures and Partitions), "Nothing significant seems to be known in general about" this quantity.



      http://www-math.mit.edu/~rstan/pubs/pubfiles/9.pdf#page=87



      The MacMahon formula for $|bf2^{(bf ktimes bf mtimes bf n)}|$ is $prod_{h=0}^{k-1}prod_{i=0}^{m-1}prod_{j=0}^{n-1}frac{h+i+j+2}{h+i+j+1}$.



      I'd be willing to accept a nested summation.



      See page 2 of James Propp, "Generating Random Elements of Finite Distributive Lattices," Electronic Journal of Combinatorics 4 (1997), R15.
      https://www.combinatorics.org/ojs/index.php/eljc/article/view/v4i2r15/pdf#page=2



      See Theorem 3.3 on page 114 of Joel Berman and Peter Koehler, "Cardinalities of Finite Distributive Lattices," Mitteilungen aus dem Mathem. Seminar Giessen 121 (1976), 103-124.
      http://homepages.math.uic.edu/~berman/freedist.pdf#page=7






      share|cite|improve this answer











      $endgroup$





















        3












        $begingroup$

        Let $P,Q$ be posets. Let $Q^P$ denote the poset of order-preserving maps from $P$ to $Q$, where $fle g$ if $f(p)le g(p)$ for all $pin P$.



        A poset has the fixed point property if for all $fin P^P$, there exists $pin P$ such that $f(p)=p$.



        If $P$ and $Q$ have the fixed point property, does $Ptimes Q$?



        Roddy, Rutkowski, and Schroeder proved the answer is "yes" if $P$ is finite.



        If $P,Q$ are finite and non-empty, does $P^Pcong Q^Q$ imply $Pcong Q$? Duffus and Wille proved that the answer is "yes" if $P$ and $Q$ are connected.



        Dwight Duffus and Rudolf Wille, "A theorem on partially ordered sets of order-preserving mappings," Proceedings of the American Mathematical Society 76 (1979), 14-16



        Conjecture. One of these questions is important. The other may not be of interest to anyone but myself.






        share|cite|improve this answer











        $endgroup$













        • $begingroup$
          What does connected mean here? The topology induced by the order?
          $endgroup$
          – Joel Adler
          Feb 13 at 7:52












        • $begingroup$
          @JoelAdler: connected in poset theory usually means "connected Hasse diagram"
          $endgroup$
          – Sam Hopkins
          Feb 13 at 15:43










        • $begingroup$
          "Connected" means that for all x and y in the poset, there exists a natural number n and elements z_1,...,z_n such that x <= z_1 >= z_2 <= z_3 >= ... <= z_n >= y.
          $endgroup$
          – Tri
          Feb 14 at 8:09



















        2












        $begingroup$

        A ranked poset is called a symmetric chain order, or SCO, if it can partitioned into rank symmetric, saturated chains. Such posets are particularly nice, as they are rank-symmetric, unimodal, and have the strong Sperner property. If $P$ and $Q$ are SCO's, then so is $Ptimes Q$. In particular, the Boolean poset ${bf 2}^n$ of subsets of ${1,2,dots,n}$, which is the product of $n$ copies of a two-element chain, is an SCO.



        Let $G$ be a subgroup embedded in the symmetric group $S_n$. This induces an action on ${bf 2}^n$ where $gS={g(s):sin S}$. Define the quotient poset ${bf 2}^n/G$ on the orbits of this action, with $[S]le [T]$ if some $S'subseteq T'$ for $S'in[S]$ and $T'in [T]$.



        Several familiar posets are realized as quotients of ${bf 2}^n$.




        • $L(m,k)$, the sub-poset of Young's lattice consisting of partitions which fit in an $mtimes k$ box. Here, $n=mk$, and the subgroup is the wreath product $S_mwr S_k$.


        • the poset of isomorphism classes finite simple graphs on $n$ vertices with the subgraph relation.




        Conjecture: For all $nge 0$ and all $Gle S_n$, ${bf 2}^n/G$ is an SCO.




        Partial results:




        • $L(4,n)$ is an SCO.


        • Letting $G=langle (1,2,dots,n)rangle$, then the necklace poset ${bf 2}^n/G$ is an SCO.







        share|cite|improve this answer











        $endgroup$













          Your Answer





          StackExchange.ifUsing("editor", function () {
          return StackExchange.using("mathjaxEditing", function () {
          StackExchange.MarkdownEditor.creationCallbacks.add(function (editor, postfix) {
          StackExchange.mathjaxEditing.prepareWmdForMathJax(editor, postfix, [["$", "$"], ["\\(","\\)"]]);
          });
          });
          }, "mathjax-editing");

          StackExchange.ready(function() {
          var channelOptions = {
          tags: "".split(" "),
          id: "504"
          };
          initTagRenderer("".split(" "), "".split(" "), channelOptions);

          StackExchange.using("externalEditor", function() {
          // Have to fire editor after snippets, if snippets enabled
          if (StackExchange.settings.snippets.snippetsEnabled) {
          StackExchange.using("snippets", function() {
          createEditor();
          });
          }
          else {
          createEditor();
          }
          });

          function createEditor() {
          StackExchange.prepareEditor({
          heartbeatType: 'answer',
          autoActivateHeartbeat: false,
          convertImagesToLinks: true,
          noModals: true,
          showLowRepImageUploadWarning: true,
          reputationToPostImages: 10,
          bindNavPrevention: true,
          postfix: "",
          imageUploader: {
          brandingHtml: "Powered by u003ca class="icon-imgur-white" href="https://imgur.com/"u003eu003c/au003e",
          contentPolicyHtml: "User contributions licensed under u003ca href="https://creativecommons.org/licenses/by-sa/3.0/"u003ecc by-sa 3.0 with attribution requiredu003c/au003e u003ca href="https://stackoverflow.com/legal/content-policy"u003e(content policy)u003c/au003e",
          allowUrls: true
          },
          noCode: true, onDemand: true,
          discardSelector: ".discard-answer"
          ,immediatelyShowMarkdownHelp:true
          });


          }
          });














          draft saved

          draft discarded


















          StackExchange.ready(
          function () {
          StackExchange.openid.initPostLogin('.new-post-login', 'https%3a%2f%2fmathoverflow.net%2fquestions%2f322598%2fopen-questions-about-posets%23new-answer', 'question_page');
          }
          );

          Post as a guest















          Required, but never shown

























          10 Answers
          10






          active

          oldest

          votes








          10 Answers
          10






          active

          oldest

          votes









          active

          oldest

          votes






          active

          oldest

          votes









          21












          $begingroup$

          The 1/3-2/3 conjecture is probably considered one of the most significant open problems about finite posets; see the Wikipedia page: https://en.wikipedia.org/wiki/1/3%E2%80%932/3_conjecture.






          share|cite|improve this answer











          $endgroup$


















            21












            $begingroup$

            The 1/3-2/3 conjecture is probably considered one of the most significant open problems about finite posets; see the Wikipedia page: https://en.wikipedia.org/wiki/1/3%E2%80%932/3_conjecture.






            share|cite|improve this answer











            $endgroup$
















              21












              21








              21





              $begingroup$

              The 1/3-2/3 conjecture is probably considered one of the most significant open problems about finite posets; see the Wikipedia page: https://en.wikipedia.org/wiki/1/3%E2%80%932/3_conjecture.






              share|cite|improve this answer











              $endgroup$



              The 1/3-2/3 conjecture is probably considered one of the most significant open problems about finite posets; see the Wikipedia page: https://en.wikipedia.org/wiki/1/3%E2%80%932/3_conjecture.







              share|cite|improve this answer














              share|cite|improve this answer



              share|cite|improve this answer








              answered Feb 6 at 17:56


























              community wiki





              Sam Hopkins
























                  16












                  $begingroup$

                  Here's a problem that I believe has little chance of being resolved, and it's also not so clear to me what the motivation behind the problem is, but it involves some very pretty algebraic combinatorics.



                  Stanley (in his PhD thesis) defined a poset $P$ to be Gaussian if for every $mgeq 0$ we have
                  $$ sum_{I in J(Ptimes [m])} q^{#I} = frac{prod_{i=1}^{r}(1-q^{h_i+m})}{prod_{i=1}^{r}(1-q^{h_i})}$$
                  where $r$ and $h_1,ldots,h_r$ are constants independent of $m$. (Here $J(P times [m])$ is the distributive lattice of order ideals of the Cartesian product of $P$ and the chain poset $[m]={1,2,ldots,m}$.)



                  Using results from Standard Monomial Theory, in "Bruhat Lattices, Plane Partition Generating Functions, and Minuscule Representations", Proctor proved that if $P$ is a minuscule poset, then $P$ is Gaussian. Here a minuscule poset is a certain finite poset coming from a minuscule representation of a semisimple Lie algebra; these have been classified- see Proctor's paper for the list. Furthermore, Proctor conjectured that the minuscule posets are the only Gaussian posets.



                  I think a few things are known (due to some mixture of Proctor and Stanley) about this conjecture: that a Gaussian poset $P$ is a disjoint union of connected Gaussian posets; that a connected Gaussian poset has to be graded; that $r$ has to be the number of elements of $P$; that the $h_i$ have to be the ranks of the elements $pin P$; maybe some more things too. But like I said I think the full conjecture is pretty hopeless.






                  share|cite|improve this answer











                  $endgroup$









                  • 10




                    $begingroup$
                    I don't think that the definition of a Gaussian poset is due to Proctor.
                    $endgroup$
                    – Richard Stanley
                    Feb 6 at 18:59






                  • 3




                    $begingroup$
                    @RichardStanley: apologies, Richard!
                    $endgroup$
                    – Sam Hopkins
                    Feb 6 at 19:00










                  • $begingroup$
                    @SamHopkins : Reading between the lines, I wonder if you suspect that the conjecture may be false but the only counterexamples are "sporadic"?
                    $endgroup$
                    – Timothy Chow
                    Feb 7 at 21:12










                  • $begingroup$
                    @TimothyChow: I think it could be false and in that case perhaps a large computer search could turn up a counterexample. If it’s true, it might just be true “by accident,” which would make it very hard to prove.
                    $endgroup$
                    – Sam Hopkins
                    Feb 7 at 21:16
















                  16












                  $begingroup$

                  Here's a problem that I believe has little chance of being resolved, and it's also not so clear to me what the motivation behind the problem is, but it involves some very pretty algebraic combinatorics.



                  Stanley (in his PhD thesis) defined a poset $P$ to be Gaussian if for every $mgeq 0$ we have
                  $$ sum_{I in J(Ptimes [m])} q^{#I} = frac{prod_{i=1}^{r}(1-q^{h_i+m})}{prod_{i=1}^{r}(1-q^{h_i})}$$
                  where $r$ and $h_1,ldots,h_r$ are constants independent of $m$. (Here $J(P times [m])$ is the distributive lattice of order ideals of the Cartesian product of $P$ and the chain poset $[m]={1,2,ldots,m}$.)



                  Using results from Standard Monomial Theory, in "Bruhat Lattices, Plane Partition Generating Functions, and Minuscule Representations", Proctor proved that if $P$ is a minuscule poset, then $P$ is Gaussian. Here a minuscule poset is a certain finite poset coming from a minuscule representation of a semisimple Lie algebra; these have been classified- see Proctor's paper for the list. Furthermore, Proctor conjectured that the minuscule posets are the only Gaussian posets.



                  I think a few things are known (due to some mixture of Proctor and Stanley) about this conjecture: that a Gaussian poset $P$ is a disjoint union of connected Gaussian posets; that a connected Gaussian poset has to be graded; that $r$ has to be the number of elements of $P$; that the $h_i$ have to be the ranks of the elements $pin P$; maybe some more things too. But like I said I think the full conjecture is pretty hopeless.






                  share|cite|improve this answer











                  $endgroup$









                  • 10




                    $begingroup$
                    I don't think that the definition of a Gaussian poset is due to Proctor.
                    $endgroup$
                    – Richard Stanley
                    Feb 6 at 18:59






                  • 3




                    $begingroup$
                    @RichardStanley: apologies, Richard!
                    $endgroup$
                    – Sam Hopkins
                    Feb 6 at 19:00










                  • $begingroup$
                    @SamHopkins : Reading between the lines, I wonder if you suspect that the conjecture may be false but the only counterexamples are "sporadic"?
                    $endgroup$
                    – Timothy Chow
                    Feb 7 at 21:12










                  • $begingroup$
                    @TimothyChow: I think it could be false and in that case perhaps a large computer search could turn up a counterexample. If it’s true, it might just be true “by accident,” which would make it very hard to prove.
                    $endgroup$
                    – Sam Hopkins
                    Feb 7 at 21:16














                  16












                  16








                  16





                  $begingroup$

                  Here's a problem that I believe has little chance of being resolved, and it's also not so clear to me what the motivation behind the problem is, but it involves some very pretty algebraic combinatorics.



                  Stanley (in his PhD thesis) defined a poset $P$ to be Gaussian if for every $mgeq 0$ we have
                  $$ sum_{I in J(Ptimes [m])} q^{#I} = frac{prod_{i=1}^{r}(1-q^{h_i+m})}{prod_{i=1}^{r}(1-q^{h_i})}$$
                  where $r$ and $h_1,ldots,h_r$ are constants independent of $m$. (Here $J(P times [m])$ is the distributive lattice of order ideals of the Cartesian product of $P$ and the chain poset $[m]={1,2,ldots,m}$.)



                  Using results from Standard Monomial Theory, in "Bruhat Lattices, Plane Partition Generating Functions, and Minuscule Representations", Proctor proved that if $P$ is a minuscule poset, then $P$ is Gaussian. Here a minuscule poset is a certain finite poset coming from a minuscule representation of a semisimple Lie algebra; these have been classified- see Proctor's paper for the list. Furthermore, Proctor conjectured that the minuscule posets are the only Gaussian posets.



                  I think a few things are known (due to some mixture of Proctor and Stanley) about this conjecture: that a Gaussian poset $P$ is a disjoint union of connected Gaussian posets; that a connected Gaussian poset has to be graded; that $r$ has to be the number of elements of $P$; that the $h_i$ have to be the ranks of the elements $pin P$; maybe some more things too. But like I said I think the full conjecture is pretty hopeless.






                  share|cite|improve this answer











                  $endgroup$



                  Here's a problem that I believe has little chance of being resolved, and it's also not so clear to me what the motivation behind the problem is, but it involves some very pretty algebraic combinatorics.



                  Stanley (in his PhD thesis) defined a poset $P$ to be Gaussian if for every $mgeq 0$ we have
                  $$ sum_{I in J(Ptimes [m])} q^{#I} = frac{prod_{i=1}^{r}(1-q^{h_i+m})}{prod_{i=1}^{r}(1-q^{h_i})}$$
                  where $r$ and $h_1,ldots,h_r$ are constants independent of $m$. (Here $J(P times [m])$ is the distributive lattice of order ideals of the Cartesian product of $P$ and the chain poset $[m]={1,2,ldots,m}$.)



                  Using results from Standard Monomial Theory, in "Bruhat Lattices, Plane Partition Generating Functions, and Minuscule Representations", Proctor proved that if $P$ is a minuscule poset, then $P$ is Gaussian. Here a minuscule poset is a certain finite poset coming from a minuscule representation of a semisimple Lie algebra; these have been classified- see Proctor's paper for the list. Furthermore, Proctor conjectured that the minuscule posets are the only Gaussian posets.



                  I think a few things are known (due to some mixture of Proctor and Stanley) about this conjecture: that a Gaussian poset $P$ is a disjoint union of connected Gaussian posets; that a connected Gaussian poset has to be graded; that $r$ has to be the number of elements of $P$; that the $h_i$ have to be the ranks of the elements $pin P$; maybe some more things too. But like I said I think the full conjecture is pretty hopeless.







                  share|cite|improve this answer














                  share|cite|improve this answer



                  share|cite|improve this answer








                  edited Feb 6 at 19:01


























                  community wiki





                  Sam Hopkins









                  • 10




                    $begingroup$
                    I don't think that the definition of a Gaussian poset is due to Proctor.
                    $endgroup$
                    – Richard Stanley
                    Feb 6 at 18:59






                  • 3




                    $begingroup$
                    @RichardStanley: apologies, Richard!
                    $endgroup$
                    – Sam Hopkins
                    Feb 6 at 19:00










                  • $begingroup$
                    @SamHopkins : Reading between the lines, I wonder if you suspect that the conjecture may be false but the only counterexamples are "sporadic"?
                    $endgroup$
                    – Timothy Chow
                    Feb 7 at 21:12










                  • $begingroup$
                    @TimothyChow: I think it could be false and in that case perhaps a large computer search could turn up a counterexample. If it’s true, it might just be true “by accident,” which would make it very hard to prove.
                    $endgroup$
                    – Sam Hopkins
                    Feb 7 at 21:16














                  • 10




                    $begingroup$
                    I don't think that the definition of a Gaussian poset is due to Proctor.
                    $endgroup$
                    – Richard Stanley
                    Feb 6 at 18:59






                  • 3




                    $begingroup$
                    @RichardStanley: apologies, Richard!
                    $endgroup$
                    – Sam Hopkins
                    Feb 6 at 19:00










                  • $begingroup$
                    @SamHopkins : Reading between the lines, I wonder if you suspect that the conjecture may be false but the only counterexamples are "sporadic"?
                    $endgroup$
                    – Timothy Chow
                    Feb 7 at 21:12










                  • $begingroup$
                    @TimothyChow: I think it could be false and in that case perhaps a large computer search could turn up a counterexample. If it’s true, it might just be true “by accident,” which would make it very hard to prove.
                    $endgroup$
                    – Sam Hopkins
                    Feb 7 at 21:16








                  10




                  10




                  $begingroup$
                  I don't think that the definition of a Gaussian poset is due to Proctor.
                  $endgroup$
                  – Richard Stanley
                  Feb 6 at 18:59




                  $begingroup$
                  I don't think that the definition of a Gaussian poset is due to Proctor.
                  $endgroup$
                  – Richard Stanley
                  Feb 6 at 18:59




                  3




                  3




                  $begingroup$
                  @RichardStanley: apologies, Richard!
                  $endgroup$
                  – Sam Hopkins
                  Feb 6 at 19:00




                  $begingroup$
                  @RichardStanley: apologies, Richard!
                  $endgroup$
                  – Sam Hopkins
                  Feb 6 at 19:00












                  $begingroup$
                  @SamHopkins : Reading between the lines, I wonder if you suspect that the conjecture may be false but the only counterexamples are "sporadic"?
                  $endgroup$
                  – Timothy Chow
                  Feb 7 at 21:12




                  $begingroup$
                  @SamHopkins : Reading between the lines, I wonder if you suspect that the conjecture may be false but the only counterexamples are "sporadic"?
                  $endgroup$
                  – Timothy Chow
                  Feb 7 at 21:12












                  $begingroup$
                  @TimothyChow: I think it could be false and in that case perhaps a large computer search could turn up a counterexample. If it’s true, it might just be true “by accident,” which would make it very hard to prove.
                  $endgroup$
                  – Sam Hopkins
                  Feb 7 at 21:16




                  $begingroup$
                  @TimothyChow: I think it could be false and in that case perhaps a large computer search could turn up a counterexample. If it’s true, it might just be true “by accident,” which would make it very hard to prove.
                  $endgroup$
                  – Sam Hopkins
                  Feb 7 at 21:16











                  11












                  $begingroup$

                  Here's another classic from algebraic combinatorics.



                  In his PhD thesis "Ordered Structures and Partitions", Stanley introduces the $(P,omega)$-partition generating function of a labeled poset $(P,omega)$. This is defined to be $K(P,omega) := sum_{sigma in A^r(P,omega)} x^{sigma}$, where $A^r(P,omega)$ is the set of all reverse $(P,omega)$-partitions (i.e., fillings of the poset $P$ with entries in $mathbb{N}$ obeying certain inequalities depending on $omega$ and the order structure of $P$), and where we use the notation $x^f := prod_{i geq 1} x_i^{#f^{-1}(i)}$. Stanley shows that $K(P,omega)$ is always a quasisymmetric function. When $P$ is the poset of a skew Young diagram and $omega$ is a compatible ``Schur labelling'' then $K_{(P,omega)}$ is in fact a symmetric function (namely, a skew Schur funciton). Stanley's conjecture is that $K_{(P,omega)}$ is a symmetric function if and only if $(P,omega)$ is isomorphic to $(P_{lambda/mu},w)$, where $lambda/mu$ is a skew-shape and $w$ is a Schur labelling of $P_{lambda/mu}$. There has been some progress on this conjecture due to Malvenuto; I don't know the exact status of what has been proved.






                  share|cite|improve this answer











                  $endgroup$









                  • 1




                    $begingroup$
                    For Malvenuto's result see link.springer.com/article/10.1007/BF01195328.
                    $endgroup$
                    – Richard Stanley
                    Feb 8 at 1:22
















                  11












                  $begingroup$

                  Here's another classic from algebraic combinatorics.



                  In his PhD thesis "Ordered Structures and Partitions", Stanley introduces the $(P,omega)$-partition generating function of a labeled poset $(P,omega)$. This is defined to be $K(P,omega) := sum_{sigma in A^r(P,omega)} x^{sigma}$, where $A^r(P,omega)$ is the set of all reverse $(P,omega)$-partitions (i.e., fillings of the poset $P$ with entries in $mathbb{N}$ obeying certain inequalities depending on $omega$ and the order structure of $P$), and where we use the notation $x^f := prod_{i geq 1} x_i^{#f^{-1}(i)}$. Stanley shows that $K(P,omega)$ is always a quasisymmetric function. When $P$ is the poset of a skew Young diagram and $omega$ is a compatible ``Schur labelling'' then $K_{(P,omega)}$ is in fact a symmetric function (namely, a skew Schur funciton). Stanley's conjecture is that $K_{(P,omega)}$ is a symmetric function if and only if $(P,omega)$ is isomorphic to $(P_{lambda/mu},w)$, where $lambda/mu$ is a skew-shape and $w$ is a Schur labelling of $P_{lambda/mu}$. There has been some progress on this conjecture due to Malvenuto; I don't know the exact status of what has been proved.






                  share|cite|improve this answer











                  $endgroup$









                  • 1




                    $begingroup$
                    For Malvenuto's result see link.springer.com/article/10.1007/BF01195328.
                    $endgroup$
                    – Richard Stanley
                    Feb 8 at 1:22














                  11












                  11








                  11





                  $begingroup$

                  Here's another classic from algebraic combinatorics.



                  In his PhD thesis "Ordered Structures and Partitions", Stanley introduces the $(P,omega)$-partition generating function of a labeled poset $(P,omega)$. This is defined to be $K(P,omega) := sum_{sigma in A^r(P,omega)} x^{sigma}$, where $A^r(P,omega)$ is the set of all reverse $(P,omega)$-partitions (i.e., fillings of the poset $P$ with entries in $mathbb{N}$ obeying certain inequalities depending on $omega$ and the order structure of $P$), and where we use the notation $x^f := prod_{i geq 1} x_i^{#f^{-1}(i)}$. Stanley shows that $K(P,omega)$ is always a quasisymmetric function. When $P$ is the poset of a skew Young diagram and $omega$ is a compatible ``Schur labelling'' then $K_{(P,omega)}$ is in fact a symmetric function (namely, a skew Schur funciton). Stanley's conjecture is that $K_{(P,omega)}$ is a symmetric function if and only if $(P,omega)$ is isomorphic to $(P_{lambda/mu},w)$, where $lambda/mu$ is a skew-shape and $w$ is a Schur labelling of $P_{lambda/mu}$. There has been some progress on this conjecture due to Malvenuto; I don't know the exact status of what has been proved.






                  share|cite|improve this answer











                  $endgroup$



                  Here's another classic from algebraic combinatorics.



                  In his PhD thesis "Ordered Structures and Partitions", Stanley introduces the $(P,omega)$-partition generating function of a labeled poset $(P,omega)$. This is defined to be $K(P,omega) := sum_{sigma in A^r(P,omega)} x^{sigma}$, where $A^r(P,omega)$ is the set of all reverse $(P,omega)$-partitions (i.e., fillings of the poset $P$ with entries in $mathbb{N}$ obeying certain inequalities depending on $omega$ and the order structure of $P$), and where we use the notation $x^f := prod_{i geq 1} x_i^{#f^{-1}(i)}$. Stanley shows that $K(P,omega)$ is always a quasisymmetric function. When $P$ is the poset of a skew Young diagram and $omega$ is a compatible ``Schur labelling'' then $K_{(P,omega)}$ is in fact a symmetric function (namely, a skew Schur funciton). Stanley's conjecture is that $K_{(P,omega)}$ is a symmetric function if and only if $(P,omega)$ is isomorphic to $(P_{lambda/mu},w)$, where $lambda/mu$ is a skew-shape and $w$ is a Schur labelling of $P_{lambda/mu}$. There has been some progress on this conjecture due to Malvenuto; I don't know the exact status of what has been proved.







                  share|cite|improve this answer














                  share|cite|improve this answer



                  share|cite|improve this answer








                  answered Feb 6 at 19:26


























                  community wiki





                  Sam Hopkins









                  • 1




                    $begingroup$
                    For Malvenuto's result see link.springer.com/article/10.1007/BF01195328.
                    $endgroup$
                    – Richard Stanley
                    Feb 8 at 1:22














                  • 1




                    $begingroup$
                    For Malvenuto's result see link.springer.com/article/10.1007/BF01195328.
                    $endgroup$
                    – Richard Stanley
                    Feb 8 at 1:22








                  1




                  1




                  $begingroup$
                  For Malvenuto's result see link.springer.com/article/10.1007/BF01195328.
                  $endgroup$
                  – Richard Stanley
                  Feb 8 at 1:22




                  $begingroup$
                  For Malvenuto's result see link.springer.com/article/10.1007/BF01195328.
                  $endgroup$
                  – Richard Stanley
                  Feb 8 at 1:22











                  6












                  $begingroup$

                  What about the Stanley–Stembridge conjecture on (3+1)-free posets? Given a poset $P$ with vertex set $V = {1,2,ldots,n}$, define the symmetric function $X_P$ in countably many indeterminates $x_1, x_2, ldots$ by
                  $$X_P := sum_{kappa:Vtomathbb{N}} x_{kappa(1)}x_{kappa(2)}cdots x_{kappa(n)}$$
                  where the sum is over all maps $kappa:V tomathbb{N}$ such that the pre-image $kappa^{-1}(i)$ of every $iinmathbb{N}$ is a totally ordered subset of $P$. Then the conjecture is that if $P$ is (3+1)-free (i.e., that $P$ contains no induced subposet isomorphic to the disjoint union of a 3-element chain and a 1-element chain) then the expansion of $X_P$ in terms of elementary symmetric functions has nonnegative coefficients.



                  This conjecture grew out of certain positivity conjectures about immanants.
                  Guay-Paquet has reduced the conjecture to the case of unit interval orders (i.e., posets that are both (3+1)-free and (2+2)-free), and the unit interval order case has a graded generalization (due to Shareshian and Wachs), which has close connections with representation theory and Hessenberg varieties.






                  share|cite|improve this answer











                  $endgroup$


















                    6












                    $begingroup$

                    What about the Stanley–Stembridge conjecture on (3+1)-free posets? Given a poset $P$ with vertex set $V = {1,2,ldots,n}$, define the symmetric function $X_P$ in countably many indeterminates $x_1, x_2, ldots$ by
                    $$X_P := sum_{kappa:Vtomathbb{N}} x_{kappa(1)}x_{kappa(2)}cdots x_{kappa(n)}$$
                    where the sum is over all maps $kappa:V tomathbb{N}$ such that the pre-image $kappa^{-1}(i)$ of every $iinmathbb{N}$ is a totally ordered subset of $P$. Then the conjecture is that if $P$ is (3+1)-free (i.e., that $P$ contains no induced subposet isomorphic to the disjoint union of a 3-element chain and a 1-element chain) then the expansion of $X_P$ in terms of elementary symmetric functions has nonnegative coefficients.



                    This conjecture grew out of certain positivity conjectures about immanants.
                    Guay-Paquet has reduced the conjecture to the case of unit interval orders (i.e., posets that are both (3+1)-free and (2+2)-free), and the unit interval order case has a graded generalization (due to Shareshian and Wachs), which has close connections with representation theory and Hessenberg varieties.






                    share|cite|improve this answer











                    $endgroup$
















                      6












                      6








                      6





                      $begingroup$

                      What about the Stanley–Stembridge conjecture on (3+1)-free posets? Given a poset $P$ with vertex set $V = {1,2,ldots,n}$, define the symmetric function $X_P$ in countably many indeterminates $x_1, x_2, ldots$ by
                      $$X_P := sum_{kappa:Vtomathbb{N}} x_{kappa(1)}x_{kappa(2)}cdots x_{kappa(n)}$$
                      where the sum is over all maps $kappa:V tomathbb{N}$ such that the pre-image $kappa^{-1}(i)$ of every $iinmathbb{N}$ is a totally ordered subset of $P$. Then the conjecture is that if $P$ is (3+1)-free (i.e., that $P$ contains no induced subposet isomorphic to the disjoint union of a 3-element chain and a 1-element chain) then the expansion of $X_P$ in terms of elementary symmetric functions has nonnegative coefficients.



                      This conjecture grew out of certain positivity conjectures about immanants.
                      Guay-Paquet has reduced the conjecture to the case of unit interval orders (i.e., posets that are both (3+1)-free and (2+2)-free), and the unit interval order case has a graded generalization (due to Shareshian and Wachs), which has close connections with representation theory and Hessenberg varieties.






                      share|cite|improve this answer











                      $endgroup$



                      What about the Stanley–Stembridge conjecture on (3+1)-free posets? Given a poset $P$ with vertex set $V = {1,2,ldots,n}$, define the symmetric function $X_P$ in countably many indeterminates $x_1, x_2, ldots$ by
                      $$X_P := sum_{kappa:Vtomathbb{N}} x_{kappa(1)}x_{kappa(2)}cdots x_{kappa(n)}$$
                      where the sum is over all maps $kappa:V tomathbb{N}$ such that the pre-image $kappa^{-1}(i)$ of every $iinmathbb{N}$ is a totally ordered subset of $P$. Then the conjecture is that if $P$ is (3+1)-free (i.e., that $P$ contains no induced subposet isomorphic to the disjoint union of a 3-element chain and a 1-element chain) then the expansion of $X_P$ in terms of elementary symmetric functions has nonnegative coefficients.



                      This conjecture grew out of certain positivity conjectures about immanants.
                      Guay-Paquet has reduced the conjecture to the case of unit interval orders (i.e., posets that are both (3+1)-free and (2+2)-free), and the unit interval order case has a graded generalization (due to Shareshian and Wachs), which has close connections with representation theory and Hessenberg varieties.







                      share|cite|improve this answer














                      share|cite|improve this answer



                      share|cite|improve this answer








                      answered Feb 7 at 16:11


























                      community wiki





                      Timothy Chow
























                          6












                          $begingroup$

                          Let $P$ be a (not necessarily finite) poset such that each element covers and is covered by a finite number of elements. Then we can define the operators $U,Dcolon mathbb{Q}Ptomathbb{Q}P$ on the vector space of formal linear combinations of elements of $P$ by $U(p) = sum_{plessdot q} q$ and $D(p) = sum_{qlessdot p}q$. Such a poset $P$ is called $r$-differential if it is locally finite, graded, with a unique minimal element, and these up and down operators satisfy $DU-UD=rI$ (where $I$ is the identity map). See https://en.wikipedia.org/wiki/Differential_poset.



                          The two prominent examples of $1$-differential posets are Young's lattice and the Young-Fibonacci lattice. It is known that these are the only $1$-differential lattices, although this was only proved relatively recently by Byrnes (https://conservancy.umn.edu/handle/11299/142992). The open problem is: are all $r$-differential lattices products of Young's lattice and the Young-Fibonacci lattice?






                          share|cite|improve this answer











                          $endgroup$













                          • $begingroup$
                            Artistic rendering of a differential lattice by Mrs. Marlena Hewitt: facebook.com/ARTMARLENA/photos/a.320409601327165/…
                            $endgroup$
                            – Tri
                            Feb 9 at 23:39


















                          6












                          $begingroup$

                          Let $P$ be a (not necessarily finite) poset such that each element covers and is covered by a finite number of elements. Then we can define the operators $U,Dcolon mathbb{Q}Ptomathbb{Q}P$ on the vector space of formal linear combinations of elements of $P$ by $U(p) = sum_{plessdot q} q$ and $D(p) = sum_{qlessdot p}q$. Such a poset $P$ is called $r$-differential if it is locally finite, graded, with a unique minimal element, and these up and down operators satisfy $DU-UD=rI$ (where $I$ is the identity map). See https://en.wikipedia.org/wiki/Differential_poset.



                          The two prominent examples of $1$-differential posets are Young's lattice and the Young-Fibonacci lattice. It is known that these are the only $1$-differential lattices, although this was only proved relatively recently by Byrnes (https://conservancy.umn.edu/handle/11299/142992). The open problem is: are all $r$-differential lattices products of Young's lattice and the Young-Fibonacci lattice?






                          share|cite|improve this answer











                          $endgroup$













                          • $begingroup$
                            Artistic rendering of a differential lattice by Mrs. Marlena Hewitt: facebook.com/ARTMARLENA/photos/a.320409601327165/…
                            $endgroup$
                            – Tri
                            Feb 9 at 23:39
















                          6












                          6








                          6





                          $begingroup$

                          Let $P$ be a (not necessarily finite) poset such that each element covers and is covered by a finite number of elements. Then we can define the operators $U,Dcolon mathbb{Q}Ptomathbb{Q}P$ on the vector space of formal linear combinations of elements of $P$ by $U(p) = sum_{plessdot q} q$ and $D(p) = sum_{qlessdot p}q$. Such a poset $P$ is called $r$-differential if it is locally finite, graded, with a unique minimal element, and these up and down operators satisfy $DU-UD=rI$ (where $I$ is the identity map). See https://en.wikipedia.org/wiki/Differential_poset.



                          The two prominent examples of $1$-differential posets are Young's lattice and the Young-Fibonacci lattice. It is known that these are the only $1$-differential lattices, although this was only proved relatively recently by Byrnes (https://conservancy.umn.edu/handle/11299/142992). The open problem is: are all $r$-differential lattices products of Young's lattice and the Young-Fibonacci lattice?






                          share|cite|improve this answer











                          $endgroup$



                          Let $P$ be a (not necessarily finite) poset such that each element covers and is covered by a finite number of elements. Then we can define the operators $U,Dcolon mathbb{Q}Ptomathbb{Q}P$ on the vector space of formal linear combinations of elements of $P$ by $U(p) = sum_{plessdot q} q$ and $D(p) = sum_{qlessdot p}q$. Such a poset $P$ is called $r$-differential if it is locally finite, graded, with a unique minimal element, and these up and down operators satisfy $DU-UD=rI$ (where $I$ is the identity map). See https://en.wikipedia.org/wiki/Differential_poset.



                          The two prominent examples of $1$-differential posets are Young's lattice and the Young-Fibonacci lattice. It is known that these are the only $1$-differential lattices, although this was only proved relatively recently by Byrnes (https://conservancy.umn.edu/handle/11299/142992). The open problem is: are all $r$-differential lattices products of Young's lattice and the Young-Fibonacci lattice?







                          share|cite|improve this answer














                          share|cite|improve this answer



                          share|cite|improve this answer








                          edited Feb 7 at 16:37


























                          community wiki





                          2 revs, 2 users 92%
                          Sam Hopkins













                          • $begingroup$
                            Artistic rendering of a differential lattice by Mrs. Marlena Hewitt: facebook.com/ARTMARLENA/photos/a.320409601327165/…
                            $endgroup$
                            – Tri
                            Feb 9 at 23:39




















                          • $begingroup$
                            Artistic rendering of a differential lattice by Mrs. Marlena Hewitt: facebook.com/ARTMARLENA/photos/a.320409601327165/…
                            $endgroup$
                            – Tri
                            Feb 9 at 23:39


















                          $begingroup$
                          Artistic rendering of a differential lattice by Mrs. Marlena Hewitt: facebook.com/ARTMARLENA/photos/a.320409601327165/…
                          $endgroup$
                          – Tri
                          Feb 9 at 23:39






                          $begingroup$
                          Artistic rendering of a differential lattice by Mrs. Marlena Hewitt: facebook.com/ARTMARLENA/photos/a.320409601327165/…
                          $endgroup$
                          – Tri
                          Feb 9 at 23:39













                          4












                          $begingroup$

                          Let $f : mathbb{F}_2^n rightarrow mathbb{F}_2$ be a boolean function. Ordering $mathbb{F}_2$ so that $0<1$, we say that $f$ is monotone if $f(x_1,ldots,x_n) le f(y_1, ldots, y_n)$ whenever $x_1 le y_1, ldots, x_n le y_n$. The Dedekind number $M(n)$ is the number of monotone Boolean functions of $n$ variables, up to logical equivalence.



                          Equivalently, $M(n)$ is the number of Boolean functions (up to logical equivalence) that can be expressed without negation, or the number of access structures on $n$ people. For example,



                          $$(x_1 wedge x_2) vee (x_1 wedge x_3) vee (x_2 wedge x_3)$$



                          is counted by $M(3)$, and corresponds to the access structure where any pair of people, or all three people, form an authorized set. In general the minimal authorized sets form an antichain in the poset of subsets of ${1,ldots, n}$, so $M(n)$ is the number the number of antichains in $mathcal{P}({1,ldots,n})$. Since the maximal subsets in an abstract simplicial complex form an antichain, $M(n)$ is also the number of abstract simplicial complexes on ${1,ldots, n}$.



                          The antichain interpretation makes it plausible that



                          $$log M(n) sim binom{n}{frac{n}{2}} sim sqrt{frac{2}{pi}} frac{2^n}{sqrt{n}} $$



                          and this was proved by Kleitman.



                          An important open problem (mentioned in the comments above) is to give a more precise asymptotic estimate for $M(n)$, and to determine the value of $M(n)$ for small $n$. At the moment, $M(n)$ is known precisely only for $n le 7$. The linked paper has the best known asymptotic formula, due to Korshunov.






                          share|cite|improve this answer











                          $endgroup$









                          • 2




                            $begingroup$
                            Small correction: $M(8)$ is known, see OEIS sequence A000372 and Wiedemann, D.: A computation of the eighth Dedekind number (Order, March 1991, Vol. 8, Issue 1, pp 5-6).
                            $endgroup$
                            – nomadictype
                            Feb 8 at 3:37










                          • $begingroup$
                            I have an English translation of Dr. Korshunov's hundred-page paper. (Even he does not have an English translation.)
                            $endgroup$
                            – Tri
                            Feb 8 at 20:49










                          • $begingroup$
                            I believe it was Yamamoto proved that if n is even, then M(n) is even. What can we say about n odd? Using an idea I have long forgotten, I think Ciucu and Stembridge got some data for n up to 19, with one exception (ether n=11 or n=13).
                            $endgroup$
                            – Tri
                            Feb 9 at 23:59
















                          4












                          $begingroup$

                          Let $f : mathbb{F}_2^n rightarrow mathbb{F}_2$ be a boolean function. Ordering $mathbb{F}_2$ so that $0<1$, we say that $f$ is monotone if $f(x_1,ldots,x_n) le f(y_1, ldots, y_n)$ whenever $x_1 le y_1, ldots, x_n le y_n$. The Dedekind number $M(n)$ is the number of monotone Boolean functions of $n$ variables, up to logical equivalence.



                          Equivalently, $M(n)$ is the number of Boolean functions (up to logical equivalence) that can be expressed without negation, or the number of access structures on $n$ people. For example,



                          $$(x_1 wedge x_2) vee (x_1 wedge x_3) vee (x_2 wedge x_3)$$



                          is counted by $M(3)$, and corresponds to the access structure where any pair of people, or all three people, form an authorized set. In general the minimal authorized sets form an antichain in the poset of subsets of ${1,ldots, n}$, so $M(n)$ is the number the number of antichains in $mathcal{P}({1,ldots,n})$. Since the maximal subsets in an abstract simplicial complex form an antichain, $M(n)$ is also the number of abstract simplicial complexes on ${1,ldots, n}$.



                          The antichain interpretation makes it plausible that



                          $$log M(n) sim binom{n}{frac{n}{2}} sim sqrt{frac{2}{pi}} frac{2^n}{sqrt{n}} $$



                          and this was proved by Kleitman.



                          An important open problem (mentioned in the comments above) is to give a more precise asymptotic estimate for $M(n)$, and to determine the value of $M(n)$ for small $n$. At the moment, $M(n)$ is known precisely only for $n le 7$. The linked paper has the best known asymptotic formula, due to Korshunov.






                          share|cite|improve this answer











                          $endgroup$









                          • 2




                            $begingroup$
                            Small correction: $M(8)$ is known, see OEIS sequence A000372 and Wiedemann, D.: A computation of the eighth Dedekind number (Order, March 1991, Vol. 8, Issue 1, pp 5-6).
                            $endgroup$
                            – nomadictype
                            Feb 8 at 3:37










                          • $begingroup$
                            I have an English translation of Dr. Korshunov's hundred-page paper. (Even he does not have an English translation.)
                            $endgroup$
                            – Tri
                            Feb 8 at 20:49










                          • $begingroup$
                            I believe it was Yamamoto proved that if n is even, then M(n) is even. What can we say about n odd? Using an idea I have long forgotten, I think Ciucu and Stembridge got some data for n up to 19, with one exception (ether n=11 or n=13).
                            $endgroup$
                            – Tri
                            Feb 9 at 23:59














                          4












                          4








                          4





                          $begingroup$

                          Let $f : mathbb{F}_2^n rightarrow mathbb{F}_2$ be a boolean function. Ordering $mathbb{F}_2$ so that $0<1$, we say that $f$ is monotone if $f(x_1,ldots,x_n) le f(y_1, ldots, y_n)$ whenever $x_1 le y_1, ldots, x_n le y_n$. The Dedekind number $M(n)$ is the number of monotone Boolean functions of $n$ variables, up to logical equivalence.



                          Equivalently, $M(n)$ is the number of Boolean functions (up to logical equivalence) that can be expressed without negation, or the number of access structures on $n$ people. For example,



                          $$(x_1 wedge x_2) vee (x_1 wedge x_3) vee (x_2 wedge x_3)$$



                          is counted by $M(3)$, and corresponds to the access structure where any pair of people, or all three people, form an authorized set. In general the minimal authorized sets form an antichain in the poset of subsets of ${1,ldots, n}$, so $M(n)$ is the number the number of antichains in $mathcal{P}({1,ldots,n})$. Since the maximal subsets in an abstract simplicial complex form an antichain, $M(n)$ is also the number of abstract simplicial complexes on ${1,ldots, n}$.



                          The antichain interpretation makes it plausible that



                          $$log M(n) sim binom{n}{frac{n}{2}} sim sqrt{frac{2}{pi}} frac{2^n}{sqrt{n}} $$



                          and this was proved by Kleitman.



                          An important open problem (mentioned in the comments above) is to give a more precise asymptotic estimate for $M(n)$, and to determine the value of $M(n)$ for small $n$. At the moment, $M(n)$ is known precisely only for $n le 7$. The linked paper has the best known asymptotic formula, due to Korshunov.






                          share|cite|improve this answer











                          $endgroup$



                          Let $f : mathbb{F}_2^n rightarrow mathbb{F}_2$ be a boolean function. Ordering $mathbb{F}_2$ so that $0<1$, we say that $f$ is monotone if $f(x_1,ldots,x_n) le f(y_1, ldots, y_n)$ whenever $x_1 le y_1, ldots, x_n le y_n$. The Dedekind number $M(n)$ is the number of monotone Boolean functions of $n$ variables, up to logical equivalence.



                          Equivalently, $M(n)$ is the number of Boolean functions (up to logical equivalence) that can be expressed without negation, or the number of access structures on $n$ people. For example,



                          $$(x_1 wedge x_2) vee (x_1 wedge x_3) vee (x_2 wedge x_3)$$



                          is counted by $M(3)$, and corresponds to the access structure where any pair of people, or all three people, form an authorized set. In general the minimal authorized sets form an antichain in the poset of subsets of ${1,ldots, n}$, so $M(n)$ is the number the number of antichains in $mathcal{P}({1,ldots,n})$. Since the maximal subsets in an abstract simplicial complex form an antichain, $M(n)$ is also the number of abstract simplicial complexes on ${1,ldots, n}$.



                          The antichain interpretation makes it plausible that



                          $$log M(n) sim binom{n}{frac{n}{2}} sim sqrt{frac{2}{pi}} frac{2^n}{sqrt{n}} $$



                          and this was proved by Kleitman.



                          An important open problem (mentioned in the comments above) is to give a more precise asymptotic estimate for $M(n)$, and to determine the value of $M(n)$ for small $n$. At the moment, $M(n)$ is known precisely only for $n le 7$. The linked paper has the best known asymptotic formula, due to Korshunov.







                          share|cite|improve this answer














                          share|cite|improve this answer



                          share|cite|improve this answer








                          edited Feb 7 at 13:50


























                          community wiki





                          3 revs
                          Mark Wildon









                          • 2




                            $begingroup$
                            Small correction: $M(8)$ is known, see OEIS sequence A000372 and Wiedemann, D.: A computation of the eighth Dedekind number (Order, March 1991, Vol. 8, Issue 1, pp 5-6).
                            $endgroup$
                            – nomadictype
                            Feb 8 at 3:37










                          • $begingroup$
                            I have an English translation of Dr. Korshunov's hundred-page paper. (Even he does not have an English translation.)
                            $endgroup$
                            – Tri
                            Feb 8 at 20:49










                          • $begingroup$
                            I believe it was Yamamoto proved that if n is even, then M(n) is even. What can we say about n odd? Using an idea I have long forgotten, I think Ciucu and Stembridge got some data for n up to 19, with one exception (ether n=11 or n=13).
                            $endgroup$
                            – Tri
                            Feb 9 at 23:59














                          • 2




                            $begingroup$
                            Small correction: $M(8)$ is known, see OEIS sequence A000372 and Wiedemann, D.: A computation of the eighth Dedekind number (Order, March 1991, Vol. 8, Issue 1, pp 5-6).
                            $endgroup$
                            – nomadictype
                            Feb 8 at 3:37










                          • $begingroup$
                            I have an English translation of Dr. Korshunov's hundred-page paper. (Even he does not have an English translation.)
                            $endgroup$
                            – Tri
                            Feb 8 at 20:49










                          • $begingroup$
                            I believe it was Yamamoto proved that if n is even, then M(n) is even. What can we say about n odd? Using an idea I have long forgotten, I think Ciucu and Stembridge got some data for n up to 19, with one exception (ether n=11 or n=13).
                            $endgroup$
                            – Tri
                            Feb 9 at 23:59








                          2




                          2




                          $begingroup$
                          Small correction: $M(8)$ is known, see OEIS sequence A000372 and Wiedemann, D.: A computation of the eighth Dedekind number (Order, March 1991, Vol. 8, Issue 1, pp 5-6).
                          $endgroup$
                          – nomadictype
                          Feb 8 at 3:37




                          $begingroup$
                          Small correction: $M(8)$ is known, see OEIS sequence A000372 and Wiedemann, D.: A computation of the eighth Dedekind number (Order, March 1991, Vol. 8, Issue 1, pp 5-6).
                          $endgroup$
                          – nomadictype
                          Feb 8 at 3:37












                          $begingroup$
                          I have an English translation of Dr. Korshunov's hundred-page paper. (Even he does not have an English translation.)
                          $endgroup$
                          – Tri
                          Feb 8 at 20:49




                          $begingroup$
                          I have an English translation of Dr. Korshunov's hundred-page paper. (Even he does not have an English translation.)
                          $endgroup$
                          – Tri
                          Feb 8 at 20:49












                          $begingroup$
                          I believe it was Yamamoto proved that if n is even, then M(n) is even. What can we say about n odd? Using an idea I have long forgotten, I think Ciucu and Stembridge got some data for n up to 19, with one exception (ether n=11 or n=13).
                          $endgroup$
                          – Tri
                          Feb 9 at 23:59




                          $begingroup$
                          I believe it was Yamamoto proved that if n is even, then M(n) is even. What can we say about n odd? Using an idea I have long forgotten, I think Ciucu and Stembridge got some data for n up to 19, with one exception (ether n=11 or n=13).
                          $endgroup$
                          – Tri
                          Feb 9 at 23:59











                          4












                          $begingroup$

                          The fish-scale conjecture: In every poset not containing an infinite antichain there exist a chain $C$ and a decomposition of the vertex set into antichains $A_i$, such that $Ccap A_i neq emptyset$ for all $iin I$. See Conjecture 10.1 in this article by Ron Aharoni and Eli Berger.



                          Interestingly, this conjecture is implied by the following covering problem in hypergraphs: Let $H=(V,E)$ be a hypergraph with the following properties:





                          1. $bigcup E = V$,

                          2. Any subset of $ein E$ is a member of $E$, and

                          3. whenever $Ssubseteq V$ and every $2$-element subset of $S$ belongs to $E$, then $Sin E$.


                          Then there is $Jsubseteq E$ such that whenever $Ksubseteq E$ has the property that $bigcup K = V$ then $|Jsetminus K| leq |Ksetminus J|$. (In that case $J$ is said to be a "strongly minimal cover" because it covers $V$ "more efficiently" in some sense, than any other cover.)



                          This covering problem is mentioned in Conjecture 10.3.(2) of the same paper.






                          share|cite|improve this answer











                          $endgroup$













                          • $begingroup$
                            I think the linked article might be wrong.
                            $endgroup$
                            – Sam Hopkins
                            Feb 8 at 15:50










                          • $begingroup$
                            Why do you think that?
                            $endgroup$
                            – Dominic van der Zypen
                            Feb 8 at 19:25










                          • $begingroup$
                            there doesn't seem to be a Conjecture 10.1 and there are 3 authors, not 2.
                            $endgroup$
                            – Sam Hopkins
                            Feb 8 at 19:30






                          • 1




                            $begingroup$
                            It looks like the correct paper is this one: math.haifa.ac.il/berger/menger_2nd_submition.pdf
                            $endgroup$
                            – Sam Hopkins
                            Feb 8 at 20:22










                          • $begingroup$
                            Thanks for providing this link!
                            $endgroup$
                            – Dominic van der Zypen
                            Feb 8 at 20:56
















                          4












                          $begingroup$

                          The fish-scale conjecture: In every poset not containing an infinite antichain there exist a chain $C$ and a decomposition of the vertex set into antichains $A_i$, such that $Ccap A_i neq emptyset$ for all $iin I$. See Conjecture 10.1 in this article by Ron Aharoni and Eli Berger.



                          Interestingly, this conjecture is implied by the following covering problem in hypergraphs: Let $H=(V,E)$ be a hypergraph with the following properties:





                          1. $bigcup E = V$,

                          2. Any subset of $ein E$ is a member of $E$, and

                          3. whenever $Ssubseteq V$ and every $2$-element subset of $S$ belongs to $E$, then $Sin E$.


                          Then there is $Jsubseteq E$ such that whenever $Ksubseteq E$ has the property that $bigcup K = V$ then $|Jsetminus K| leq |Ksetminus J|$. (In that case $J$ is said to be a "strongly minimal cover" because it covers $V$ "more efficiently" in some sense, than any other cover.)



                          This covering problem is mentioned in Conjecture 10.3.(2) of the same paper.






                          share|cite|improve this answer











                          $endgroup$













                          • $begingroup$
                            I think the linked article might be wrong.
                            $endgroup$
                            – Sam Hopkins
                            Feb 8 at 15:50










                          • $begingroup$
                            Why do you think that?
                            $endgroup$
                            – Dominic van der Zypen
                            Feb 8 at 19:25










                          • $begingroup$
                            there doesn't seem to be a Conjecture 10.1 and there are 3 authors, not 2.
                            $endgroup$
                            – Sam Hopkins
                            Feb 8 at 19:30






                          • 1




                            $begingroup$
                            It looks like the correct paper is this one: math.haifa.ac.il/berger/menger_2nd_submition.pdf
                            $endgroup$
                            – Sam Hopkins
                            Feb 8 at 20:22










                          • $begingroup$
                            Thanks for providing this link!
                            $endgroup$
                            – Dominic van der Zypen
                            Feb 8 at 20:56














                          4












                          4








                          4





                          $begingroup$

                          The fish-scale conjecture: In every poset not containing an infinite antichain there exist a chain $C$ and a decomposition of the vertex set into antichains $A_i$, such that $Ccap A_i neq emptyset$ for all $iin I$. See Conjecture 10.1 in this article by Ron Aharoni and Eli Berger.



                          Interestingly, this conjecture is implied by the following covering problem in hypergraphs: Let $H=(V,E)$ be a hypergraph with the following properties:





                          1. $bigcup E = V$,

                          2. Any subset of $ein E$ is a member of $E$, and

                          3. whenever $Ssubseteq V$ and every $2$-element subset of $S$ belongs to $E$, then $Sin E$.


                          Then there is $Jsubseteq E$ such that whenever $Ksubseteq E$ has the property that $bigcup K = V$ then $|Jsetminus K| leq |Ksetminus J|$. (In that case $J$ is said to be a "strongly minimal cover" because it covers $V$ "more efficiently" in some sense, than any other cover.)



                          This covering problem is mentioned in Conjecture 10.3.(2) of the same paper.






                          share|cite|improve this answer











                          $endgroup$



                          The fish-scale conjecture: In every poset not containing an infinite antichain there exist a chain $C$ and a decomposition of the vertex set into antichains $A_i$, such that $Ccap A_i neq emptyset$ for all $iin I$. See Conjecture 10.1 in this article by Ron Aharoni and Eli Berger.



                          Interestingly, this conjecture is implied by the following covering problem in hypergraphs: Let $H=(V,E)$ be a hypergraph with the following properties:





                          1. $bigcup E = V$,

                          2. Any subset of $ein E$ is a member of $E$, and

                          3. whenever $Ssubseteq V$ and every $2$-element subset of $S$ belongs to $E$, then $Sin E$.


                          Then there is $Jsubseteq E$ such that whenever $Ksubseteq E$ has the property that $bigcup K = V$ then $|Jsetminus K| leq |Ksetminus J|$. (In that case $J$ is said to be a "strongly minimal cover" because it covers $V$ "more efficiently" in some sense, than any other cover.)



                          This covering problem is mentioned in Conjecture 10.3.(2) of the same paper.







                          share|cite|improve this answer














                          share|cite|improve this answer



                          share|cite|improve this answer








                          answered Feb 8 at 11:44


























                          community wiki





                          Dominic van der Zypen













                          • $begingroup$
                            I think the linked article might be wrong.
                            $endgroup$
                            – Sam Hopkins
                            Feb 8 at 15:50










                          • $begingroup$
                            Why do you think that?
                            $endgroup$
                            – Dominic van der Zypen
                            Feb 8 at 19:25










                          • $begingroup$
                            there doesn't seem to be a Conjecture 10.1 and there are 3 authors, not 2.
                            $endgroup$
                            – Sam Hopkins
                            Feb 8 at 19:30






                          • 1




                            $begingroup$
                            It looks like the correct paper is this one: math.haifa.ac.il/berger/menger_2nd_submition.pdf
                            $endgroup$
                            – Sam Hopkins
                            Feb 8 at 20:22










                          • $begingroup$
                            Thanks for providing this link!
                            $endgroup$
                            – Dominic van der Zypen
                            Feb 8 at 20:56


















                          • $begingroup$
                            I think the linked article might be wrong.
                            $endgroup$
                            – Sam Hopkins
                            Feb 8 at 15:50










                          • $begingroup$
                            Why do you think that?
                            $endgroup$
                            – Dominic van der Zypen
                            Feb 8 at 19:25










                          • $begingroup$
                            there doesn't seem to be a Conjecture 10.1 and there are 3 authors, not 2.
                            $endgroup$
                            – Sam Hopkins
                            Feb 8 at 19:30






                          • 1




                            $begingroup$
                            It looks like the correct paper is this one: math.haifa.ac.il/berger/menger_2nd_submition.pdf
                            $endgroup$
                            – Sam Hopkins
                            Feb 8 at 20:22










                          • $begingroup$
                            Thanks for providing this link!
                            $endgroup$
                            – Dominic van der Zypen
                            Feb 8 at 20:56
















                          $begingroup$
                          I think the linked article might be wrong.
                          $endgroup$
                          – Sam Hopkins
                          Feb 8 at 15:50




                          $begingroup$
                          I think the linked article might be wrong.
                          $endgroup$
                          – Sam Hopkins
                          Feb 8 at 15:50












                          $begingroup$
                          Why do you think that?
                          $endgroup$
                          – Dominic van der Zypen
                          Feb 8 at 19:25




                          $begingroup$
                          Why do you think that?
                          $endgroup$
                          – Dominic van der Zypen
                          Feb 8 at 19:25












                          $begingroup$
                          there doesn't seem to be a Conjecture 10.1 and there are 3 authors, not 2.
                          $endgroup$
                          – Sam Hopkins
                          Feb 8 at 19:30




                          $begingroup$
                          there doesn't seem to be a Conjecture 10.1 and there are 3 authors, not 2.
                          $endgroup$
                          – Sam Hopkins
                          Feb 8 at 19:30




                          1




                          1




                          $begingroup$
                          It looks like the correct paper is this one: math.haifa.ac.il/berger/menger_2nd_submition.pdf
                          $endgroup$
                          – Sam Hopkins
                          Feb 8 at 20:22




                          $begingroup$
                          It looks like the correct paper is this one: math.haifa.ac.il/berger/menger_2nd_submition.pdf
                          $endgroup$
                          – Sam Hopkins
                          Feb 8 at 20:22












                          $begingroup$
                          Thanks for providing this link!
                          $endgroup$
                          – Dominic van der Zypen
                          Feb 8 at 20:56




                          $begingroup$
                          Thanks for providing this link!
                          $endgroup$
                          – Dominic van der Zypen
                          Feb 8 at 20:56











                          3












                          $begingroup$

                          Find a nice expression for the number of down-sets (order ideals) of a product of four finite chains (totally ordered sets), $|bf2^{(bf ktimes bf mtimes bf ntimes bf r)}|$, where $k,m,n,r$ are positive integers and "$bf k$" denotes the $k$-element chain, ${0,1,dots,k-1}$.



                          Richard P. Stanley writes (on page 83 of Ordered Structures and Partitions), "Nothing significant seems to be known in general about" this quantity.



                          http://www-math.mit.edu/~rstan/pubs/pubfiles/9.pdf#page=87



                          The MacMahon formula for $|bf2^{(bf ktimes bf mtimes bf n)}|$ is $prod_{h=0}^{k-1}prod_{i=0}^{m-1}prod_{j=0}^{n-1}frac{h+i+j+2}{h+i+j+1}$.



                          I'd be willing to accept a nested summation.



                          See page 2 of James Propp, "Generating Random Elements of Finite Distributive Lattices," Electronic Journal of Combinatorics 4 (1997), R15.
                          https://www.combinatorics.org/ojs/index.php/eljc/article/view/v4i2r15/pdf#page=2



                          See Theorem 3.3 on page 114 of Joel Berman and Peter Koehler, "Cardinalities of Finite Distributive Lattices," Mitteilungen aus dem Mathem. Seminar Giessen 121 (1976), 103-124.
                          http://homepages.math.uic.edu/~berman/freedist.pdf#page=7






                          share|cite|improve this answer











                          $endgroup$


















                            3












                            $begingroup$

                            Find a nice expression for the number of down-sets (order ideals) of a product of four finite chains (totally ordered sets), $|bf2^{(bf ktimes bf mtimes bf ntimes bf r)}|$, where $k,m,n,r$ are positive integers and "$bf k$" denotes the $k$-element chain, ${0,1,dots,k-1}$.



                            Richard P. Stanley writes (on page 83 of Ordered Structures and Partitions), "Nothing significant seems to be known in general about" this quantity.



                            http://www-math.mit.edu/~rstan/pubs/pubfiles/9.pdf#page=87



                            The MacMahon formula for $|bf2^{(bf ktimes bf mtimes bf n)}|$ is $prod_{h=0}^{k-1}prod_{i=0}^{m-1}prod_{j=0}^{n-1}frac{h+i+j+2}{h+i+j+1}$.



                            I'd be willing to accept a nested summation.



                            See page 2 of James Propp, "Generating Random Elements of Finite Distributive Lattices," Electronic Journal of Combinatorics 4 (1997), R15.
                            https://www.combinatorics.org/ojs/index.php/eljc/article/view/v4i2r15/pdf#page=2



                            See Theorem 3.3 on page 114 of Joel Berman and Peter Koehler, "Cardinalities of Finite Distributive Lattices," Mitteilungen aus dem Mathem. Seminar Giessen 121 (1976), 103-124.
                            http://homepages.math.uic.edu/~berman/freedist.pdf#page=7






                            share|cite|improve this answer











                            $endgroup$
















                              3












                              3








                              3





                              $begingroup$

                              Find a nice expression for the number of down-sets (order ideals) of a product of four finite chains (totally ordered sets), $|bf2^{(bf ktimes bf mtimes bf ntimes bf r)}|$, where $k,m,n,r$ are positive integers and "$bf k$" denotes the $k$-element chain, ${0,1,dots,k-1}$.



                              Richard P. Stanley writes (on page 83 of Ordered Structures and Partitions), "Nothing significant seems to be known in general about" this quantity.



                              http://www-math.mit.edu/~rstan/pubs/pubfiles/9.pdf#page=87



                              The MacMahon formula for $|bf2^{(bf ktimes bf mtimes bf n)}|$ is $prod_{h=0}^{k-1}prod_{i=0}^{m-1}prod_{j=0}^{n-1}frac{h+i+j+2}{h+i+j+1}$.



                              I'd be willing to accept a nested summation.



                              See page 2 of James Propp, "Generating Random Elements of Finite Distributive Lattices," Electronic Journal of Combinatorics 4 (1997), R15.
                              https://www.combinatorics.org/ojs/index.php/eljc/article/view/v4i2r15/pdf#page=2



                              See Theorem 3.3 on page 114 of Joel Berman and Peter Koehler, "Cardinalities of Finite Distributive Lattices," Mitteilungen aus dem Mathem. Seminar Giessen 121 (1976), 103-124.
                              http://homepages.math.uic.edu/~berman/freedist.pdf#page=7






                              share|cite|improve this answer











                              $endgroup$



                              Find a nice expression for the number of down-sets (order ideals) of a product of four finite chains (totally ordered sets), $|bf2^{(bf ktimes bf mtimes bf ntimes bf r)}|$, where $k,m,n,r$ are positive integers and "$bf k$" denotes the $k$-element chain, ${0,1,dots,k-1}$.



                              Richard P. Stanley writes (on page 83 of Ordered Structures and Partitions), "Nothing significant seems to be known in general about" this quantity.



                              http://www-math.mit.edu/~rstan/pubs/pubfiles/9.pdf#page=87



                              The MacMahon formula for $|bf2^{(bf ktimes bf mtimes bf n)}|$ is $prod_{h=0}^{k-1}prod_{i=0}^{m-1}prod_{j=0}^{n-1}frac{h+i+j+2}{h+i+j+1}$.



                              I'd be willing to accept a nested summation.



                              See page 2 of James Propp, "Generating Random Elements of Finite Distributive Lattices," Electronic Journal of Combinatorics 4 (1997), R15.
                              https://www.combinatorics.org/ojs/index.php/eljc/article/view/v4i2r15/pdf#page=2



                              See Theorem 3.3 on page 114 of Joel Berman and Peter Koehler, "Cardinalities of Finite Distributive Lattices," Mitteilungen aus dem Mathem. Seminar Giessen 121 (1976), 103-124.
                              http://homepages.math.uic.edu/~berman/freedist.pdf#page=7







                              share|cite|improve this answer














                              share|cite|improve this answer



                              share|cite|improve this answer








                              answered Feb 9 at 17:37


























                              community wiki





                              Tri
























                                  3












                                  $begingroup$

                                  Let $P,Q$ be posets. Let $Q^P$ denote the poset of order-preserving maps from $P$ to $Q$, where $fle g$ if $f(p)le g(p)$ for all $pin P$.



                                  A poset has the fixed point property if for all $fin P^P$, there exists $pin P$ such that $f(p)=p$.



                                  If $P$ and $Q$ have the fixed point property, does $Ptimes Q$?



                                  Roddy, Rutkowski, and Schroeder proved the answer is "yes" if $P$ is finite.



                                  If $P,Q$ are finite and non-empty, does $P^Pcong Q^Q$ imply $Pcong Q$? Duffus and Wille proved that the answer is "yes" if $P$ and $Q$ are connected.



                                  Dwight Duffus and Rudolf Wille, "A theorem on partially ordered sets of order-preserving mappings," Proceedings of the American Mathematical Society 76 (1979), 14-16



                                  Conjecture. One of these questions is important. The other may not be of interest to anyone but myself.






                                  share|cite|improve this answer











                                  $endgroup$













                                  • $begingroup$
                                    What does connected mean here? The topology induced by the order?
                                    $endgroup$
                                    – Joel Adler
                                    Feb 13 at 7:52












                                  • $begingroup$
                                    @JoelAdler: connected in poset theory usually means "connected Hasse diagram"
                                    $endgroup$
                                    – Sam Hopkins
                                    Feb 13 at 15:43










                                  • $begingroup$
                                    "Connected" means that for all x and y in the poset, there exists a natural number n and elements z_1,...,z_n such that x <= z_1 >= z_2 <= z_3 >= ... <= z_n >= y.
                                    $endgroup$
                                    – Tri
                                    Feb 14 at 8:09
















                                  3












                                  $begingroup$

                                  Let $P,Q$ be posets. Let $Q^P$ denote the poset of order-preserving maps from $P$ to $Q$, where $fle g$ if $f(p)le g(p)$ for all $pin P$.



                                  A poset has the fixed point property if for all $fin P^P$, there exists $pin P$ such that $f(p)=p$.



                                  If $P$ and $Q$ have the fixed point property, does $Ptimes Q$?



                                  Roddy, Rutkowski, and Schroeder proved the answer is "yes" if $P$ is finite.



                                  If $P,Q$ are finite and non-empty, does $P^Pcong Q^Q$ imply $Pcong Q$? Duffus and Wille proved that the answer is "yes" if $P$ and $Q$ are connected.



                                  Dwight Duffus and Rudolf Wille, "A theorem on partially ordered sets of order-preserving mappings," Proceedings of the American Mathematical Society 76 (1979), 14-16



                                  Conjecture. One of these questions is important. The other may not be of interest to anyone but myself.






                                  share|cite|improve this answer











                                  $endgroup$













                                  • $begingroup$
                                    What does connected mean here? The topology induced by the order?
                                    $endgroup$
                                    – Joel Adler
                                    Feb 13 at 7:52












                                  • $begingroup$
                                    @JoelAdler: connected in poset theory usually means "connected Hasse diagram"
                                    $endgroup$
                                    – Sam Hopkins
                                    Feb 13 at 15:43










                                  • $begingroup$
                                    "Connected" means that for all x and y in the poset, there exists a natural number n and elements z_1,...,z_n such that x <= z_1 >= z_2 <= z_3 >= ... <= z_n >= y.
                                    $endgroup$
                                    – Tri
                                    Feb 14 at 8:09














                                  3












                                  3








                                  3





                                  $begingroup$

                                  Let $P,Q$ be posets. Let $Q^P$ denote the poset of order-preserving maps from $P$ to $Q$, where $fle g$ if $f(p)le g(p)$ for all $pin P$.



                                  A poset has the fixed point property if for all $fin P^P$, there exists $pin P$ such that $f(p)=p$.



                                  If $P$ and $Q$ have the fixed point property, does $Ptimes Q$?



                                  Roddy, Rutkowski, and Schroeder proved the answer is "yes" if $P$ is finite.



                                  If $P,Q$ are finite and non-empty, does $P^Pcong Q^Q$ imply $Pcong Q$? Duffus and Wille proved that the answer is "yes" if $P$ and $Q$ are connected.



                                  Dwight Duffus and Rudolf Wille, "A theorem on partially ordered sets of order-preserving mappings," Proceedings of the American Mathematical Society 76 (1979), 14-16



                                  Conjecture. One of these questions is important. The other may not be of interest to anyone but myself.






                                  share|cite|improve this answer











                                  $endgroup$



                                  Let $P,Q$ be posets. Let $Q^P$ denote the poset of order-preserving maps from $P$ to $Q$, where $fle g$ if $f(p)le g(p)$ for all $pin P$.



                                  A poset has the fixed point property if for all $fin P^P$, there exists $pin P$ such that $f(p)=p$.



                                  If $P$ and $Q$ have the fixed point property, does $Ptimes Q$?



                                  Roddy, Rutkowski, and Schroeder proved the answer is "yes" if $P$ is finite.



                                  If $P,Q$ are finite and non-empty, does $P^Pcong Q^Q$ imply $Pcong Q$? Duffus and Wille proved that the answer is "yes" if $P$ and $Q$ are connected.



                                  Dwight Duffus and Rudolf Wille, "A theorem on partially ordered sets of order-preserving mappings," Proceedings of the American Mathematical Society 76 (1979), 14-16



                                  Conjecture. One of these questions is important. The other may not be of interest to anyone but myself.







                                  share|cite|improve this answer














                                  share|cite|improve this answer



                                  share|cite|improve this answer








                                  edited Feb 9 at 23:49


























                                  community wiki





                                  2 revs
                                  Tri













                                  • $begingroup$
                                    What does connected mean here? The topology induced by the order?
                                    $endgroup$
                                    – Joel Adler
                                    Feb 13 at 7:52












                                  • $begingroup$
                                    @JoelAdler: connected in poset theory usually means "connected Hasse diagram"
                                    $endgroup$
                                    – Sam Hopkins
                                    Feb 13 at 15:43










                                  • $begingroup$
                                    "Connected" means that for all x and y in the poset, there exists a natural number n and elements z_1,...,z_n such that x <= z_1 >= z_2 <= z_3 >= ... <= z_n >= y.
                                    $endgroup$
                                    – Tri
                                    Feb 14 at 8:09


















                                  • $begingroup$
                                    What does connected mean here? The topology induced by the order?
                                    $endgroup$
                                    – Joel Adler
                                    Feb 13 at 7:52












                                  • $begingroup$
                                    @JoelAdler: connected in poset theory usually means "connected Hasse diagram"
                                    $endgroup$
                                    – Sam Hopkins
                                    Feb 13 at 15:43










                                  • $begingroup$
                                    "Connected" means that for all x and y in the poset, there exists a natural number n and elements z_1,...,z_n such that x <= z_1 >= z_2 <= z_3 >= ... <= z_n >= y.
                                    $endgroup$
                                    – Tri
                                    Feb 14 at 8:09
















                                  $begingroup$
                                  What does connected mean here? The topology induced by the order?
                                  $endgroup$
                                  – Joel Adler
                                  Feb 13 at 7:52






                                  $begingroup$
                                  What does connected mean here? The topology induced by the order?
                                  $endgroup$
                                  – Joel Adler
                                  Feb 13 at 7:52














                                  $begingroup$
                                  @JoelAdler: connected in poset theory usually means "connected Hasse diagram"
                                  $endgroup$
                                  – Sam Hopkins
                                  Feb 13 at 15:43




                                  $begingroup$
                                  @JoelAdler: connected in poset theory usually means "connected Hasse diagram"
                                  $endgroup$
                                  – Sam Hopkins
                                  Feb 13 at 15:43












                                  $begingroup$
                                  "Connected" means that for all x and y in the poset, there exists a natural number n and elements z_1,...,z_n such that x <= z_1 >= z_2 <= z_3 >= ... <= z_n >= y.
                                  $endgroup$
                                  – Tri
                                  Feb 14 at 8:09




                                  $begingroup$
                                  "Connected" means that for all x and y in the poset, there exists a natural number n and elements z_1,...,z_n such that x <= z_1 >= z_2 <= z_3 >= ... <= z_n >= y.
                                  $endgroup$
                                  – Tri
                                  Feb 14 at 8:09











                                  2












                                  $begingroup$

                                  A ranked poset is called a symmetric chain order, or SCO, if it can partitioned into rank symmetric, saturated chains. Such posets are particularly nice, as they are rank-symmetric, unimodal, and have the strong Sperner property. If $P$ and $Q$ are SCO's, then so is $Ptimes Q$. In particular, the Boolean poset ${bf 2}^n$ of subsets of ${1,2,dots,n}$, which is the product of $n$ copies of a two-element chain, is an SCO.



                                  Let $G$ be a subgroup embedded in the symmetric group $S_n$. This induces an action on ${bf 2}^n$ where $gS={g(s):sin S}$. Define the quotient poset ${bf 2}^n/G$ on the orbits of this action, with $[S]le [T]$ if some $S'subseteq T'$ for $S'in[S]$ and $T'in [T]$.



                                  Several familiar posets are realized as quotients of ${bf 2}^n$.




                                  • $L(m,k)$, the sub-poset of Young's lattice consisting of partitions which fit in an $mtimes k$ box. Here, $n=mk$, and the subgroup is the wreath product $S_mwr S_k$.


                                  • the poset of isomorphism classes finite simple graphs on $n$ vertices with the subgraph relation.




                                  Conjecture: For all $nge 0$ and all $Gle S_n$, ${bf 2}^n/G$ is an SCO.




                                  Partial results:




                                  • $L(4,n)$ is an SCO.


                                  • Letting $G=langle (1,2,dots,n)rangle$, then the necklace poset ${bf 2}^n/G$ is an SCO.







                                  share|cite|improve this answer











                                  $endgroup$


















                                    2












                                    $begingroup$

                                    A ranked poset is called a symmetric chain order, or SCO, if it can partitioned into rank symmetric, saturated chains. Such posets are particularly nice, as they are rank-symmetric, unimodal, and have the strong Sperner property. If $P$ and $Q$ are SCO's, then so is $Ptimes Q$. In particular, the Boolean poset ${bf 2}^n$ of subsets of ${1,2,dots,n}$, which is the product of $n$ copies of a two-element chain, is an SCO.



                                    Let $G$ be a subgroup embedded in the symmetric group $S_n$. This induces an action on ${bf 2}^n$ where $gS={g(s):sin S}$. Define the quotient poset ${bf 2}^n/G$ on the orbits of this action, with $[S]le [T]$ if some $S'subseteq T'$ for $S'in[S]$ and $T'in [T]$.



                                    Several familiar posets are realized as quotients of ${bf 2}^n$.




                                    • $L(m,k)$, the sub-poset of Young's lattice consisting of partitions which fit in an $mtimes k$ box. Here, $n=mk$, and the subgroup is the wreath product $S_mwr S_k$.


                                    • the poset of isomorphism classes finite simple graphs on $n$ vertices with the subgraph relation.




                                    Conjecture: For all $nge 0$ and all $Gle S_n$, ${bf 2}^n/G$ is an SCO.




                                    Partial results:




                                    • $L(4,n)$ is an SCO.


                                    • Letting $G=langle (1,2,dots,n)rangle$, then the necklace poset ${bf 2}^n/G$ is an SCO.







                                    share|cite|improve this answer











                                    $endgroup$
















                                      2












                                      2








                                      2





                                      $begingroup$

                                      A ranked poset is called a symmetric chain order, or SCO, if it can partitioned into rank symmetric, saturated chains. Such posets are particularly nice, as they are rank-symmetric, unimodal, and have the strong Sperner property. If $P$ and $Q$ are SCO's, then so is $Ptimes Q$. In particular, the Boolean poset ${bf 2}^n$ of subsets of ${1,2,dots,n}$, which is the product of $n$ copies of a two-element chain, is an SCO.



                                      Let $G$ be a subgroup embedded in the symmetric group $S_n$. This induces an action on ${bf 2}^n$ where $gS={g(s):sin S}$. Define the quotient poset ${bf 2}^n/G$ on the orbits of this action, with $[S]le [T]$ if some $S'subseteq T'$ for $S'in[S]$ and $T'in [T]$.



                                      Several familiar posets are realized as quotients of ${bf 2}^n$.




                                      • $L(m,k)$, the sub-poset of Young's lattice consisting of partitions which fit in an $mtimes k$ box. Here, $n=mk$, and the subgroup is the wreath product $S_mwr S_k$.


                                      • the poset of isomorphism classes finite simple graphs on $n$ vertices with the subgraph relation.




                                      Conjecture: For all $nge 0$ and all $Gle S_n$, ${bf 2}^n/G$ is an SCO.




                                      Partial results:




                                      • $L(4,n)$ is an SCO.


                                      • Letting $G=langle (1,2,dots,n)rangle$, then the necklace poset ${bf 2}^n/G$ is an SCO.







                                      share|cite|improve this answer











                                      $endgroup$



                                      A ranked poset is called a symmetric chain order, or SCO, if it can partitioned into rank symmetric, saturated chains. Such posets are particularly nice, as they are rank-symmetric, unimodal, and have the strong Sperner property. If $P$ and $Q$ are SCO's, then so is $Ptimes Q$. In particular, the Boolean poset ${bf 2}^n$ of subsets of ${1,2,dots,n}$, which is the product of $n$ copies of a two-element chain, is an SCO.



                                      Let $G$ be a subgroup embedded in the symmetric group $S_n$. This induces an action on ${bf 2}^n$ where $gS={g(s):sin S}$. Define the quotient poset ${bf 2}^n/G$ on the orbits of this action, with $[S]le [T]$ if some $S'subseteq T'$ for $S'in[S]$ and $T'in [T]$.



                                      Several familiar posets are realized as quotients of ${bf 2}^n$.




                                      • $L(m,k)$, the sub-poset of Young's lattice consisting of partitions which fit in an $mtimes k$ box. Here, $n=mk$, and the subgroup is the wreath product $S_mwr S_k$.


                                      • the poset of isomorphism classes finite simple graphs on $n$ vertices with the subgraph relation.




                                      Conjecture: For all $nge 0$ and all $Gle S_n$, ${bf 2}^n/G$ is an SCO.




                                      Partial results:




                                      • $L(4,n)$ is an SCO.


                                      • Letting $G=langle (1,2,dots,n)rangle$, then the necklace poset ${bf 2}^n/G$ is an SCO.








                                      share|cite|improve this answer














                                      share|cite|improve this answer



                                      share|cite|improve this answer








                                      answered Feb 8 at 21:08


























                                      community wiki





                                      Mike Earnest































                                          draft saved

                                          draft discarded




















































                                          Thanks for contributing an answer to MathOverflow!


                                          • Please be sure to answer the question. Provide details and share your research!

                                          But avoid



                                          • Asking for help, clarification, or responding to other answers.

                                          • Making statements based on opinion; back them up with references or personal experience.


                                          Use MathJax to format equations. MathJax reference.


                                          To learn more, see our tips on writing great answers.




                                          draft saved


                                          draft discarded














                                          StackExchange.ready(
                                          function () {
                                          StackExchange.openid.initPostLogin('.new-post-login', 'https%3a%2f%2fmathoverflow.net%2fquestions%2f322598%2fopen-questions-about-posets%23new-answer', 'question_page');
                                          }
                                          );

                                          Post as a guest















                                          Required, but never shown





















































                                          Required, but never shown














                                          Required, but never shown












                                          Required, but never shown







                                          Required, but never shown

































                                          Required, but never shown














                                          Required, but never shown












                                          Required, but never shown







                                          Required, but never shown







                                          Popular posts from this blog

                                          How do I know what Microsoft account the skydrive app is syncing to?

                                          When does type information flow backwards in C++?

                                          Grease: Live!